Download as docx, pdf, or txt
Download as docx, pdf, or txt
You are on page 1of 100

An audit of FS is conducted to determine if the

Overall financial statements are stated in accordance with an applicable financial reporting framework

Client’s internal control is functioning as intended

Organization is operating efficiently and effectively

Auditee is following specific procedures or rules set down by some higher authority
Ans.
Overall financial statements are stated in accordance with an applicable financial reporting framework

The auditor is required to maintain professional skepticism throughout the audit. Which of the following
statements concerning professional scepticism is false?

A belief that management and those charged with governance are honest and have integrity relieves the
auditor of the need to maintain professional scepticism.

Maintaining professional skepticism throughout the audit reduces the risk of using inappropriate
assumptions in determining the nature, timing, and extent of the audit procedures and evaluating the results
thereof

Professional scepticism is necessary to the critical assessment of audit evidence.

Professional scepticism is an attitude that includes questioning contradictory audit evidence obtained.
Ans.
A belief that management and those charged with governance are honest and have integrity relieves the
auditor of the need to maintain professional scepticism.

The auditor’s opinion

Enhances the credibility of the financial statements.

Is an assurance as to the future viability of the entity.

Is an assurance as to the efficiency with which management has conducted the affairs of the entity, but not
effectiveness.

Certifies the correctness of the financial statements.


Ans.
Enhances the credibility of the financial statements.

The purpose of an audit of FS is to


Enhance the knowledge and skills of external auditors
Enhance understanding of the stakeholders of the company
Enhance the degree of confidence of intended users of the FS
Provide government agencies, such as BIR with basis for assessments
Ans.
Enhance the degree of confidence of intended users of the FS

Which of the following is incorrect regarding the general principles of an audit?


The auditor should comply with the “Code of Ethics for Professional Ethics for Certified Public Accountants”
promulgated by the Philippine Professional Regulation Commission.

The auditor should conduct an audit in accordance with PSAs.

The auditor should plan and perform an audit with an attitude of professional skepticism recognizing that
circumstances may exist that cause the financial statements to be materially misstated.

The auditor would ordinarily expect to find evidence to support management representations and
assume they are necessarily correct.
Ans.
The auditor would ordinarily expect to find evidence to support management representations and
assume they are necessarily correct.

It refers to the audit procedures deemed necessary in the circumstances to achieve the objective of the audit.

Scope of an audit
Objective of an audit
Audit program
Reasonable assurance
Ans.
Scope of an audit

Management assertions are

Stated as the footnotes to the financial statements

Implied or expressed representations about the accounts in the financial statements

Explicitly expressed representations about the financial statements

Provided to the auditor in the Assertions Letter, but are not disclosed on the financial statements
Ans.
Implied or expressed representations about the accounts in the financial statements

Which of the following is not one of the seven broad categories of financial statement assertions, as classified
in Glossary of Terms?

Ownership
Existence
Valuation
Presentation and disclosure
Ans.
Ownership

The five major phases in conducting a risk- based audit process are:
a. Audit planning and risk assessment activities
b. Preliminary engagement activities
c. Completing the audit and post- audit responsibilities
d. Reporting and communication
e. Responses to assessed risks

The proper sequence in applying the above steps is:


baecd baedc beacd beadc
Ans.
Baedc

Which of the following is the correct order of steps in the audit process?
A. Perform test of controls
B. Develop an overall strategy for the expected conduct and scope of the audit
C. Obtain client’s written representation
D. Prepare engagement letter
E. Perform substantive tests
D, A, B, E, C D, B, A, E, C D, B, C, A, E D, B, E, A, C
Ans.
D, B, A, E, C

When an auditor believes that an understanding with the client has t been established, he or she should
ordinarily
Perform the audit with increased professional scepticism
Decline to accept or perform the audit
Assess control risk at the maximum level and perform a primarily substantive audit.
Modify the scope of the audit to reflect an increase of material misstatement due to fraud
Ans.
Decline to accept or perform the audit

Which of the following is not included in an engagement letter?


Restriction on cash balances, lines of credit by similar arrangements
Accessibility to all financial records
Client imposed limitation in the scope
Limitation in the scope of examination as imposed by circumstances
Ans.
Restriction on cash balances, lines of credit by similar arrangements

Assuming a recurring audit, in which of the following situations would the auditor be unlikely to send a new
engagement letter to the client?
A recent change in partner and/or staff involved in the audit engagement.
A change in the terms of engagement.
A recent change of client management.
A significant change in the nature or size of the client's business.
Ans.
A recent change in partner and/or staff involved in the audit engagement.

The understanding between the client and the auditor as to the degree of responsibilities to be assumed by
each is normally set forth in a (an)
representation letter. engagement letter management letter comfort letter
Ans.
engagement letter
In a continuing engagement, the continuing auditor would most likely send a new engagement letter when
there is a change in the partner assigned to the engagement
there is a recent change in the client’s board of directors
there are new accounting pronouncements affecting the client’s financial statements there are expected
minor changes in the nature or size of the client’s business
Ans.
there is a recent change in the client’s board of directors

An engagement letter should be written before the start of an audit because


it may limit the auditor's legal liability by specifying the auditor's responsibilities
it specifies the client's responsibility for preparing schedules and making the records available to the auditor.
it specifies the basis for billing the audit for the upcoming year.
All of the choices given
Ans.
All of the choices given

The following are the reasons why the auditor will send a new engagement letter each period, except
Any revised or special terms of the engagement
A recent change in the audit team
A significant change in nature or size of the client’s business
A change in the legal requirements
Ans.
A recent change in the audit team

Dora, CPA, accepted an engagement to audit the financial statements of Tech Resources, a nonpublic entity.
Before the completion of the audit, Tech requested Dora to change the engagement to a review of financial
statements. Before Dora agrees to change the engagement, Dora is required to consider the

(1) Legal or contractual implications of the request

(2) Reason given for Tech’s Request

(1)No; (2)No
(1)Yes; (2)Yes
(1)Yes; (2)No
(1)No; (2)Yes
Ans.
(1)Yes; (2)Yes

Prior to the acceptance of an audit engagement with a client who has terminated the services of the
predecessor auditor, the CPA should

Contact the predecessor auditor without advising the prospective client and request a complete report of the
circumstances leading to the termination with the understanding that all information disclosed will be kept
confidential
Accept the engagement without contacting the predecessor auditor since the CPA will include procedures to
verify the reason given by the client for termination
Not communicate with the predecessor auditor because this would in effect be asking the auditor to provide
the confidential relationship between the auditor and client.
Advise the client of the intention to contact the predecessor auditor and request permission for the contact.
Ans.
Advise the client of the intention to contact the predecessor auditor and request permission for the contact.

Preplanning the audit involves several key activities. Which of the following would not be included in
preplanning an audit?
Investigating the client’s background
Determining the likelihood of issuing an unqualified audit opinion on the client’s financial statements
Communicating with the prospective client’s prior auditor to inquire about any disagreements with the
client.
Understanding the client’s reasons for obtaining an audit
Ans.
Determining the likelihood of issuing an unqualified audit opinion on the client’s financial statements

Which of the following is not included in the engagement letter?


Objectives of the engagement
Representation that the financial statements were prepared in accordance with generally accepted
accounting principles
Management’s responsibilities
A clear explanation of the services to be performed on the engagement
Ans.
Representation that the financial statements were prepared in accordance with generally accepted accounting
principles

What is the most likely course of action that will be taken by an auditor in assessing management integrity?
Tour the plant
Review the minutes of the board of directors
Research the background and histories of officers
Review bank reconciliation
Ans.
Research the background and histories of officers

Which of the following factors do not influence the decision of the auditor to send a separate engagement
letter to the parent entity and its component (subsidiary, branch, or division) assuming the same auditor
handles both entities?
Legal requirements
Degree of ownership by parent
Ethical requirements
Whether a separate audit report is to be issued on the component
Ans.
Ethical requirements

The form and content of audit engagement letters may vary for each client, but they would generally include
reference to the following, except
The objective of the audit of financial statements
Auditor’s responsibility for the financial statements
The form of any reports or other communication or results of the engagements
Unrestricted access to whatever records, documentation and other information requested in connection with
the audit.
Ans.
Auditor’s responsibility for the financial statements

Which of the following would be least likely to be included in the auditor’s engagement letter?
Forms of any reports
Extent of his responsibilities to his client
Objectives and scope of the audit
Type of opinion to be issued
Ans.
Type of opinion to be issued

Which of the following will an auditor most likely discuss with the former auditors of a potential client prior
to acceptance?
Integrity management
Reasons for changing audit firms
Disagreements with management regarding accounting principles
All of the above must be discussed
Ans.
All of the above must be discussed

Engagement letter that documents and confirms the auditor’s acceptance of the engagement would normally
be sent to the client.
before the audit report is issued
after the audit report is issued
after the audit report is issued
before the commencement of the engagement
Ans.
before the commencement of the engagement

Before accepting an audit engagement, you as the successor auditor would least likely make specific inquiries
of the previous auditor regarding
Facts that might bear on the integrity of management.
The degree of cooperation the previous auditor received from the client’s lawyer.
An inquiry regarding disagreements with management as to auditing procedures.
The predecessor auditor understands as to the reasons for the change of auditors.
Ans.
The degree of cooperation the previous auditor received from the client’s lawyer.

According to PSA 210 (Terms of Engagements), which of the following is appropriately included in an
engagement letter?

(I) Because of the test nature and other inherent limitations of an audit, together with the inherent limitations
of any accounting and internal control system, there is unavoidable risk that even some material
misstatements may remain undiscovered
(II )Our audit will be made with the objective of our expressing an opinion on the financial statements

(III )An audit also includes evaluating the accounting policies used and significant estimates made by
management
I, II I, III I, III I, II, III
Ans.
I, II, III

The form and content of the audit engagement letters may vary for each client, but they would generally
include reference to:
management’s responsibility for the financial statements
restricted access to whatever records, documentation and other information requested in connection with
the audit.
the fact that because of the test nature and other inherent limitations of an audit, together with the inherent
limitations of any accounting and internal control system, there is an avoidable risk that even some material
misstatements may remain undiscovered.
the type of opinion that may be given out by the auditor.
Ans.
management’s responsibility for the financial statements

Arrangements concerning which of the following are least likely to be included in engagement letter?
Auditor’s responsibilities
Fees and billing
CPA investment in client securities
Other forms of reports to be issued in addition to the audit report
Ans.
CPA investment in client securities

When a professional accountant is the auditor of a parent entity and also the auditor of its subsidiary, branch
or division (component), which of the following factors need not be considered in deciding whether to send
the separate engagement letter to the component Who appoints the auditor of the component.
Whether a separate audit report is to be issued on the component.
Legal requirements
Number of reports to be prepared during the peak audit season.
Ans.
Number of reports to be prepared during the peak audit season.

An engagement letter should ordinarily include information on the objectives of the engagement and (CPA
responsibilities, Client responsibilities, Limitation of engagement) Yes, Yes, No Yes, No, Yes Yes,
Yes, Yes No, Yes, Yes
Ans.
Yes, Yes, Yes

On an audit engagement performed by a CPA firm with one office, at the minimum, knowledge of the relevant
professional accounting and auditing standards should be held by The auditor with final responsibility for the
audit
All professionals working upon the audit
All professionals working upon the audit and the partner in charge of the CPA firm
All professionals working in the office
Ans.
The auditor with final responsibility for the audit

Which of the following actions may be appropriate if the auditor is unable to agree to a change of the
engagement and is not permitted to continue the original engagement?

(I) Auditor should withdraw from the engagement

(II) Consider whether there is any obligation to report to the board of directors or shareholders the
circumstances necessitating withdrawal
I I, II II Neither I nor II
Ans.
I, II

The use of an engagement letter is the best method of documenting

(I) The required communication of significant deficiencies in internal control structure

(II) Significantly lower materiality levels than those used in the prior audit

(III) The description of any letters or reports that the auditor expects to issue

(IV) Arrangements concerning the involvement of internal auditors and other client staff

a. I and II I and IV II and III III and IV


Ans.
III and IV

The engagement letter will do one, some, or all of the following:


1. State whether the CPA will perform audit, review, or compilation services.
2) State whether the CPA will perform tax or management advisory or other services.
3) State any restriction to be imposed on the CPA’s work.
4) Identify deadlines for completing the work.
5) State the amount and type of work to be done by client’s personnel in generating auditor’s working
papers.
6) State the CPA’s fees for the engagement.
Inform the client that the CPA does not have responsibility for detecting fraud.

The engagement letter does

number 1, 2, 4 and 6
number 1, 2, 3, 4, and 6
number 1, 3, 5, and 7
All seven of the above stated items.
Ans.
All seven of the above stated items.
An engagement letter would not normally include
billing arrangement
arrangement concerning client’s assistance
details of the procedures that will be performed
expectation of receiving a representation letter from management
Ans.
details of the procedures that will be performed

Communication with a predecessor auditor is initiated by:


Management
The successor auditor
The audit committee of the board of directors
The chair of the board of directors
Ans.
The successor auditor

In which of the following situations would the auditor be unlikely to send a new engagement letter to a
continuing client?
a change in terms of the engagemen
a significant change in the nature or size of the client’s business
a recent change of client management
a recent change in the partner and/or staff in the audit engagement
Ans.
a recent change in the partner and/or staff in the audit engagement

What is the most likely course of action that will be taken by an auditor in assessing management integrity?
Tour the plant
Review the minutes of the board of directors
Research the background and histories of officers
Review bank reconciliation
Ans.
Research the background and histories of officers

The auditor shall agree the terms of the audit engagement with management or those charged with
governance, as appropriate. Which of the following normally signs the engagement letter for an audit of a
public company
Corporate treasurer
Chief financial officer
Chairman of the board of directors
Audit committee
Ans.
Audit committee

Which of the following would ordinarily be considered a reasonable basis for requesting a change in the
engagement?
a change in circumstances.
a misunderstanding as to the nature of the audit.
a restriction on the scope of the engagement, whether imposed by management or caused by circumstances.
Both a and b
Ans.
Both a and b

Which of the following is (are) valid reasons why an auditor sends to his client an engagement letter?

(1) To avoid misunderstanding with respect to engagement


(2) To confirm the auditor’s acceptance of the appointment
(3) To document the objective and scope of the audit
(4) To ensure CPA’s compliance to PSA

(1)Yes; (2)Yes; (3)Yes; (4)Yes;


(1)Yes; (2)Yes; (3)Yes; (4)No;
(1)No; (2)Yes; (3)Yes; (4)No;
(1)Yes; (2)No; (3)Yes; (4)Yes;
Ans.
(1)Yes; (2)Yes; (3)Yes; (4)No;

An engagement letter is best described as:

A letter from company management to the auditors specifying management’s expectations for completion of
the audit on a timely basis and the fees.
A letter from the auditors to company management is responsible for the financial statements, and the
auditors will issue an opinion on the financial statements.
A letter from the auditors to company management that specifies the responsibilities of both the company
and the auditors in completing the audit and the timing for its completion.
A letter from the Board of Directors’ audit committee to the auditor that indicates the auditor has been
engaged to perform the audit and the fees to be paid.
Ans.
A letter from the auditors to company management that specifies the responsibilities of both the company
and the auditors in completing the audit and the timing for its completion.
Which of the following will an auditor least likely discuss with the former auditors of a potential client prior to
acceptance?
Integrity/of management
Fees charged for services
Disagreements with management regarding accounting principles
Reasons for changing audit firms
Ans.
Fees charged for services

According to PSA 210, the auditor and the client should agree on the terms of engagement. The agreed terms
would need to be recorded in a(n)
memorandum to be placed in the permanent section of the auditing working papers
engagement letter
client representation letter
comfort letter
Ans.
engagement letter

Which of the following is not one of the principal contents of an engagement letter?
objectives of the financial statements
unrestricted access to records and documents
limitations of the engagement
management’s responsibility for the financial statements
Ans.
objectives of the financial statements

When a CPA is approached to perform an audit for the first time, the CPA should make inquiries of the
predecessor auditor. This is necessary procedure because the predecessor may be able to provide the
successor with information that will assist the successor in determining whether the engagement should be
accepted.
Before accepting an audit engagement, a successor auditor makes specific inquiries of the predecessor
auditor regarding disagreements the predecessor had with the client concerning auditing procedures and
accounting principles.

True, False
False, True
Both statements are false
Both statements are true
Ans.
Both statements are true

Before performing any audit procedures. The auditor and the client should agree on the

(1) Type of opinion to be expressed

(2) Specific procedures to be performed

(3) Terms of the engagement


(1)Yes; (2)No; (3)Yes
(1)No; (2)Yes; (3)Yes
(1)No; (2)No; (3)Yes
(1)Yes; (2)Yes; (3)Yes
Ans.
(1)No; (2)No; (3)Yes

Retainer’s fee basis is when


Billing is done in the basis of actual time spent at the agreed rate/hours
The client is charged on a per diem basis with a cap or ceiling amount
The client is billed a fixed fee periodically for the services rendered during a designated period of time.
The client is billed at a single amount for the entire engagement
Ans.
The client is billed a fixed fee periodically for the services rendered during a designated period of time.

Prior to the acceptance of an audit engagement with a client who has terminated the services of the
predecessor auditor, the auditor should
Contact the predecessor auditor without advising the prospective client and request a complete report of the
circumstance leading to the termination with the understanding that all information disclosed will be kept
confidential.
Accept the engagement without contacting the predecessor auditor since the CPA can include audit
procedures to verify the reason given by the client for the termination.
Not communicate with the predecessor auditor because this would in effect be asking the auditor to violate
the confidential relationship between auditor and client.
Advise the client of the intention to contact the predecessor auditor and request permission for the contact.
Ans.
Advise the client of the intention to contact the predecessor auditor and request permission for the contact.

The following matters are generally included in an auditor’s engagement letter, except
Management’s responsibility for the financial statements
The scope of the audit
The fact that because of the test nature and other inherent limitations of internal control, there is an
unavoidable risk that even some material misstatements may remain undiscovered.
The factors to be considered in setting preliminary judgments about materiality
Ans.
The factors to be considered in setting preliminary judgments about materiality

Assuming the recurring audit, in which of the following situations would the auditor be unlikely to send a new
engagement letter to the client?
A recent change partner and/ or staff involved in the audit engagement.
A change revision in the terms of engagement.
A recent change of client nature or size or management
A misunderstanding as to objective and scope of audit.
Ans.
A recent change partner and/ or staff involved in the audit engagement.

The auditor would most likely withdraw from the engagement if


the auditor discovers material misstatement in the client’s financial statement
the auditor is unable to agree to a change of the engagement and is not permitted to continue the original
engagement
there are multiple uncertainties affecting the financial statements
there is substantial doubt abut entity’s ability to continue as a going concern
Ans.
the auditor is unable to agree to a change of the engagement and is not permitted to continue the original
engagement

Which of the following would not be a valid justification for changing the nature of an engagement?
there was a misunderstanding concerning the original engagement
there was a recent change in client’s requirement for audited financial statement
there was a management imposed scope limitation that requires modification of audit opinion.
the cost of completing the audit is significant and the client no longer needs audited financial statements.
Ans.
• there was a management imposed scope limitation that requires modification of audit opinion.

Which of the following is (are) valid reasons why an auditor sends to his client an engagement letter?

To avoid misunderstanding with respect to engagement


To confirm the auditor’s acceptance of the appointment
To document the objective and scope of the audit
To ensure CPA’s compliance to PSA

Yes
Yes
Yes
Yes

Yes
Yes
Yes
No

No
Yes
Yes
No

Yes
No
Yes
Yes
Ans.

Yes
Yes
Yes
No

An engagement letter should ordinarily include information on the objectives of the engagement and ….
CPA’s responsibilities Management’s responsibilities Limitations of engagement

Yes
Yes
Yes

Yes
No
Yes

Yes
No
No

No
No
No
Ans.
Yes
Yes
Yes

When an auditor increase the planned assessed level control risk because certain control procedures were
determined to be ineffective the auditor would most likely increase the
Extent of test of details (substantive tests)
Level of inherent risk
Extent of test of controls
Level of detection risk
Ans.
Extent of test of details (substantive tests)

To obtain evidential matter abut control risk, an auditor selects tests from a variety of techniques including
Inquiry Calculations Analytical procedures Confirmations
Ans.
Inquiry

For the auditor to assess control risk account balances at less than the maximum
No significant weaknesses must have occurred
The internal auditor must test and evaluate some of the controls
The external auditor must test and valuate some of the controls
Management must test and evaluate some of the controls.
Ans.
The external auditor must test and valuate some of the controls

In considering internal control, what is the purpose of a transaction walk through?


To assure that employees are performing assigned functions accurately
To confirm the auditor’s understanding of the internal control structure
To select documents for detailed tests of controls
To verify the results of the auditor’s sampling plan
Ans.
To confirm the auditor’s understanding of the internal control structure

An auditor obtains knowledge about a new client's business and its industry to
Make constructive suggestions concerning improvements to the client's internal control. Develop an attitude
of professional skepticism concerning management's financial statement assertions.
Evaluate whether the aggregation of known misstatements causes the financial statements takes as a whole
to be materially misstated.
Understand the events and transactions that may have an effect on the client's financial statements.
Ans.
Understand the events and transactions that may have an effect on the client's financial statements.

PSAs requires the auditor to obtain understanding of the entity’s internal control structure For first time
control structure
For every audit
Whenever the auditor wishes or sees necessary
Sufficient to find any frauds that may exist
Ans.
For every audit

Which of the following is not a reason that the auditor must gain an understanding of the client’s internal
control system?
To better understand the client, its risk, and how it manages those risks.
To assess control risk and identify the types of financial statement misstatements that are most likely to
occur affecting relevant financial statement assertions.
To plan direct tests of account balances to determine if misstatements have occurred. All are reasons why
auditors must gain an understanding of the client’s internal control system.
Ans.
All are reasons why auditors must gain an understanding of the client’s internal control system.

A procedure that would least likely be used by an auditor in performing tests of control is Inspection
Reperformance Observation Recalculation
Ans.
Recalculation

In the audit of a private company, the auditor will test controls when control risk is initially assessed at:

Low moderate high

yes no yes

no no yes

yes yes no

no yes no
Ans.

yes yes no

A primary objective of procedures performed to obtain an understanding of internal control is to provide an


auditor with
Knowledge necessary to assess the risks of material misstatements
Evidence to use in assessing inherent risk
A basis for modifying tests of controls
An evaluation of the consistency of application of management policies
Ans.
Knowledge necessary to assess the risks of material misstatements

When the auditor attempts to understand the operation of the accounting system by tracing a few
transactions through the accounting system, the auditor is said to be:

Tracing
Vouching
Performing a walk- through
Testing controls
Ans.
Performing a walk- through

Which of the following statements is correct with respect to obtaining an understanding with a client?

Auditors are not required to obtain an understanding with their clients.


Auditors must obtain an understanding only if an audit is to be conducted.
Auditor must document their understanding of the engagement.
Auditors must obtain an engagement letter.
Ans.
Auditor must document their understanding of the engagement.
Which of the following is not useful for obtaining an understanding of internal controls?

Make inquiries of the client’s personnel


Examine documents and records
Read industry trade magazines
Observe client activities and operations
Ans.
Read industry trade magazines

Reasons to evaluate internal control would not include

Basis for planning the audit


Determining the nature, timing, and extent of substantive procedures
Basis for type of opinion to be rendered
Formulating constructive suggestions for improvements
Ans.
Basis for type of opinion to be rendered

Before relying on the system of internal control, the auditor obtains a reasonable degree of assurance that the
internal control procedures are in use and operating as planned. The auditor obtains this assurance by
performing

Substantive tests
Transaction tests
Compliance test/ Tests of controls
Tests of trends and ratios
Ans.
Compliance test/ Tests of controls

Which of the following would an auditor least likely perform when obtaining understanding of the entity’s
internal control?

Reperformance of internal control


Inquiries of appropriate personnel
Inspection of documents and record
Observation of the entity’s activities and operations
Ans.
Reperformance of internal control

When obtaining understanding of the entity’s control, the auditor should obtain knowledge about the internal
control’s
1) Design
2) Implementation
3) Operating effectiveness
1) yes 2) yes 3) yes
1) yes 2) yes 3) no
1) yes 2) no 3) no
1) no 2) no 3) yes
Ans.
1) yes 2) yes 3) no

Adequate audit planning helps ensure that appropriate attention is devoted:


1) To important areas of the audit
2) So that potential problems are promptly identified
3) So that the work is completed expeditiously

1) Yes 2) Yes 3) No

1) Yes 2) Yes 3) Yes

1) Yes 2) No 3) No

1) Yes 2) No 3) Yes
Ans.
1) Yes 2) Yes 3) Yes

The auditor’s understanding of the entity and the environment consists of the following aspects

I. Industry, regulatory, and other external factors, including the applicable financial reporting framework
II. Nature of the entity, including the entity’s selection and application of accounting policies
III. Objectives and strategies and the related business risks that may result in a material misstatement of the
financial statements.
IV. Measurement and review of the entity’s financial performance.
Internal control

All of the above I, II and III only I, II, III and IV only I, and III only
Ans.
All of the above

Which of the following statements about auditor documentation of the client’s internal control is correct?
Documentation must include flow charts.
Documentation must include procedural write- offs.
No documentation is necessary although it is desirable
No one particular form of documentation is necessary, and the extent of documentation may vary.
Ans.
No one particular form of documentation is necessary, and the extent of documentation may vary.

In an auditor’s consideration of internal control, the completion of a questionnaire is most closely associated
with which of the following?
Separation of duties
Flowchart accuracy
Understanding the system
Tests of controls
Ans.
Understanding the system

To obtain an understanding of the relevant policies and procedures of internal control, the auditor performs
all of the following except:
Make inquiries
Make observations
Design substantive tests
Inspect documents and records
Ans.
Design substantive tests

Because of the risk of material misstatement, an audit of financial statements in accordance with PSAs should
be planned and performed with an attitude of
Impartial conservatism.
Objective judgment.
Independent integrity.
Professional skepticism.
Ans.
Professional skepticism.

The primary objective of procedures performed to obtain an understanding of internal control is to provide
an auditor with
Evidential matter to use in reducing detection risk.
A basis from which to modify tests of controls.
Knowledge necessary to plan the audit.
Information necessary to prepare flowcharts.
Ans.
Knowledge necessary to plan the audit.

After obtaining an understanding of a client’s controls, an auditor may decide to omit tests of the controls.
Which of the following in not appropriate reason to omit tests of controls? The controls duplicate other
controls.
The controls appear adequate.
Reportable conditions preclude assessing control risk below the maximum.
The effort to test controls exceeds the effort saved by not performing substantive tests.
Ans.
The controls appear adequate.

The auditor's review of the client's internal control is documented in order to substantiate Conformity of the
accounting records with PFRS.
Adherence to requirements of management.
Compliance with PSA.
The fairness of the financial statement presentation.
Ans.
Compliance with PSA.

Which of the following is correct about internal control?


Accounting and internal control systems provide management with conclusive evidence that objectives are
reached.
One of the inherent limitations of accounting and internal control systems is the possibility that the
procedures may become inadequate due to changes in conditions, and compliance with procedures may
deteriorate.
Most internal controls tend to be directed at non-routine transactions.
Management does not consider costs of the accounting and internal control systems.
Ans.
One of the inherent limitations of accounting and internal control systems is the possibility that the
procedures may become inadequate due to changes in conditions, and compliance with procedures may
deteriorate.

Of the following, the best statement of the CPA’s primary objective in considering internal control is that the
review is intended to provide
A basis for reliance on the system and determining the scope of other auditing procedures.
Reasonable protection against client fraud and defalcations by client employees.
A basis for constructive suggestions to the client for improving his internal control system.
A method for ensuring that there is reasonable assurance that the financial statements are reliable.
Ans.
A basis for reliance on the system and determining the scope of other auditing procedures.

When obtaining an understanding of the accounting and internal control system the auditor may trace a few
transactions through the accounting system. This technique is:
Reperformance test Test of transactions Walk-through test Validity test
Ans.
Walk-through test

In an auditor’s consideration of internal control, the completion of a questionnaire is most closely associated
with which of the following?
Separation of duties
Understanding the system
Flowchart accuracy
Tests of controls
Ans.
Understanding the system

Which of the following statements about internal control is correct?


Properly maintained internal controls reasonably assure that collusion among employees cannot occur.
Establishing and maintaining internal control is the internal auditor’s responsibility.
Exceptionally strong control allows the auditor to eliminate substantive tests.
The cost-benefit relationship should be considered in designing internal control.
Ans.
The cost-benefit relationship should be considered in designing internal control.

Corporate directors, management, external auditors, and internal auditors all play important roles in creating
a proper control environment. Top management is primarily responsible for
Establishing a proper environment and specifying overall internal control.
Reviewing the reliability and integrity of financial information and the means used to collect and report such
information.
Ensuring that external and internal auditors adequately monitor the control environment.
Implementing and monitoring controls designed by the board of directors.
Ans.
Establishing a proper environment and specifying overall internal control.

In obtaining an understanding of an entity’s internal control structure, an auditor is required to obtain


knowledge about the
Operating effectiveness of Design of policies
Policies and procedures and procedures

Yes Yes
No Yes
Yes No
No No
Ans.
No Yes

An auditor intends to perform tests of control on a client’s cash disbursements procedures. If the control
procedures leave no audit trail of documentary evidence, the auditor most likely will test the procedures by
Inquiry and analytical procedures.
Inquiry and observation.
Analytical procedures and confirmation.
Confirmation and observation.
Ans.
Inquiry and observation.

After gaining an understanding of internal control and assessing the risks of material misstatement, an
auditor decided to perform tests of controls. The auditor most likely decided that
Additional evidence to support a further reduction in control risk is not available.
It is not possible or practicable to reduce the risks of material misstatement at the assertion level to an
acceptably low level with audit evidence obtained only from substantive test procedures.
There were many internal control weaknesses that could allow misstatements to enter the accounting
system.
Increase in the assessed level of control risk is justified for certain financial statement assertions.
Ans.
It is not possible or practicable to reduce the risks of material misstatement at the assertion level to an
acceptably low level with audit evidence obtained only from substantive test procedures.

The following are the matters to be considered by the auditor in establishing the overall audit strategy, except
Defining the scope of the examinations
Assess risk and materiality
Computation of audit fees
Ascertaining the reporting objectives of the engagement
Ans.
Computation of audit fees

Which of the following matters should be considered by the auditor in developing the overall audit strategy?
Important characteristics of the entity, its business, its financial performance and its reporting requirements
including changes since the date of the prior audit.
Conditions requiring special attention, such as the existence of related parties.
The setting of materiality levels for audit purposes.
All of the above.
Ans.
All of the above.
When the auditor identifies a misstatement in the financial statements, the auditor should consider whether
such a misstatement may be indicative of fraud and if there is such an indication, the auditor should
Consider the implications of the misstatement in relation to other aspects of the audit. Withdraw from the
engagement.
Communicate the information to regulatory and enforcement authorities.
Report the matter to the person or persons who made the audit appointment.
Ans.
Consider the implications of the misstatement in relation to other aspects of the audit.

When planning the audit, the auditor should make inquiries of management. Such inquiries should address
the following, except
Management’s assessment of the risk that the financial statements may be misstated due to fraud.
Management’s process for identifying and responding to the risks of fraud in the entity. Management’s
consideration of how an element of unpredictability will be incorporated into the nature, timing, and extent of
the audit procedures to be performed.
Management’s communication, if any, to those charged with governance regarding its processes for
identifying and responding to the risks of fraud in the entity.
Ans.
Management’s consideration of how an element of unpredictability will be incorporated into the nature,
timing, and extent of the audit procedures to be performed.

In developing the overall audit strategy for a new client factor not to be considered is:
The terms of the engagement and any statutory responsibilities.
The client’s business, including the structure of the organization and accounting system used.
The specific procedures to be performed to gather audit evidence.
The involvement of other auditors in the audit of components of the group
Ans.
The specific procedures to be performed to gather audit evidence.

The following are the matters to be considered by the auditor in establishing the overall audit strategy, except
Defining the scope of the examinations
Assess risk and materiality
Computation of audit fees
Ascertaining the reporting objectives of the engagement
Ans.
Computation of audit fees

Which of the following is not considered by the auditor when preparing an overall audit plan?
The effect of information technology on the audit
The nature and timing of reports and other communication with the entity that are expected under the
engagement.
The identification of complex accounting areas including those involving accounting estimates.
The content of the management representation letter.
Ans.
The content of the management representation letter.

Audit risk components consist of inherent, control and detection risk. Which of them is/are dependent
variables?
Inherent risk Control risk Detection risk Inherent and control risks
Ans.
Detection risk

An auditor should consider two key issues when obtaining an understanding of a client’s internal controls.
These issues are
The effectiveness and efficiency of the controls.
The frequency and effectiveness of the controls.
The design and implementation of the controls.
The implementation and efficiency of the controls.
Ans.
The design and implementation of the controls.

The auditor is most likely to presume that a high risk of irregularities exists if
the client is a multinational company that does business in numerous foreign countries. the client does
business with several related parties.
inadequate segregation of duties places an employee in a position to perpetrate and conceal thefts.
inadequate employee training results in lengthy EDP exception report each month
Ans.
inadequate segregation of duties places an employee in a position to perpetrate and conceal thefts.

While assessing the risks of material misstatement auditors identify risks, relate risk to what could go wrong,
consider the magnitude of risks and
Assess the risk of misstatements due to illegal acts
Consider the complexity of the transactions involved
Consider the likelihood that the risks could result in material misstatements
Determine materiality levels
Ans.
Consider the likelihood that the risks could result in material misstatements

Which of the following statements best describes materiality?


Materiality is typically measured as a fixed percentage of assets.
Materiality is typically measured as a fixed percentage of net income.
Materiality does not depend on the company being audited, but is solely dependent on the auditor’s
discretion.
Materiality is the amount at which judgment based on the financial statements may be altered.
Ans.
Materiality is the amount at which judgment based on the financial statements may be altered.

As the acceptable level of detection risk increases, an auditor may change the
Timing of substantive tests by performing them at year end rather than at interim date. Nature of substantive
tests from a more effective to a less effective procedure
Timing of tests of controls by performing them at several dates rather than at one time Assessed level of
inherent risk to a higher amount
Ans.
Nature of substantive tests from a more effective to a less effective procedure

Which of the following procedures would an auditor least likely perform while obtaining an understanding of
a client in a financial statement audit?
Coordinating the assistance of entity personnel in data preparation
Discussing matters that may affect the audit with firm personnel responsible for non-audit services to the
entity
Selecting a sample of vendors’ invoices for comparison to receiving reports
Reading the current year’s interim financial statements
Ans.
Selecting a sample of vendors’ invoices for comparison to receiving reports

Which of the following audit risk components may be assessed in non-quantitative terms?
1) Inherent Risk
2) Control Risk
3) Detection Risk

1) Yes 2) Yes 3) No

1) Yes 2) No 3) Yes

1) No 2) Yes 3) Yes

1) Yes 2) Yes 3) Yes


Ans.
1) Yes 2) Yes 3) Yes

The assessment of the risks of material misstatement at the financial statement level is affected by the
auditor’s understanding of the control environment. Weaknesses in the control environment ordinarily will
lead the auditor to
Have more confidence in internal control and the reliability of audit evidence generated internally within the
entity.
Conduct some audit procedures at an interim date rather than at period end.
Modify the nature of audit procedures to obtain more persuasive audit evidence.
Decrease the number of locations to be included in the audit scope.
Ans.
Modify the nature of audit procedures to obtain more persuasive audit evidence.

After discovering that a related-party transaction exists, the auditor should be aware that the
substance of the transaction could be significantly different from its form.
adequacy of disclosure of the transaction is secondary to its legal form.
transaction is assumed to be outside the ordinary course of business.
financial statements should recognize the legal form of the transaction rather than its substance.
Ans.
substance of the transaction could be significantly different from its form.

Inherent risk and control risk differ from detection risk in that inherent risk and control risk are
Elements of audit risk while detection risk is not.
Changed at the auditor’s discretion while detection risk is not.
Considered at the individual account-balance level while detection risk is not.
Functions of the client and its environment while detection risk is not.
Ans.
Functions of the client and its environment while detection risk is not.
Which of the following is correct statement?
The auditor should use professional judgment to assess audit risk and to design audit procedures to ensure it
is eliminated.
The auditor is an insurer, and his or her report constitutes a guarantee.
The subsequent discovery that a material misstatement exists in the financial statements is evidence of
inadequate planning, performance, or judgment on the part of the auditor.
The auditor should obtain an understanding of the accounting and internal control systems sufficient to plan
the audit and develop an effective audit approach.
Ans.
The auditor should obtain an understanding of the accounting and internal control systems sufficient to plan
the audit and develop an effective audit approach.

The five steps in applying materiality are listed below in random order.

1. Estimate the combined misstatement


2. Estimate the total misstatement in segment
3. Set primarily judgment about materiality to segments
4. Allocate preliminary judgment about materiality segments
5. Compare combined estimate with preliminary judgment about materiality

1,2,5,4,3 3,4,2,1,5 4,3,1,5,2 5,1,3,2,4


Ans.
3,4,2,1,5

Which of the following conditions and events may most likely indicate the existence of risks of material,
misstatements?
Having personnel with appropriate accounting and financial reporting skills.
Accounting measurements hat involve simple processes
Significant amount of routine or systematic transactions
Constraints on the availability of capital and credit
Ans.
Constraints on the availability of capital and credit

In comparing management fraud with employee fraud, the auditor’s risk of failing to discover the fraud is
greater for employee fraud because of the larger number of employee in the organization.
greater for employee fraud because of the higher crime rate among blue collar workers. greater for
management fraud because of management ‘s ability to override existing internal controls.
greater for management fraud because managers are inherently smarter than employees.
Ans.
greater for management fraud because of management ‘s ability to override existing internal controls.

Which statement is incorrect regarding the discussion among the engagement team about the susceptibility of
the entity’s financial statements to material misstatements?
The members of the engagement team should discuss the susceptibility of the entity’s financial statements to
material misstatements.
The objective of this discussion is for members of the engagement team to gain a better understanding of the
potential for material misstatements of the financial statements resulting from fraud or error in the specific
areas assigned to them, and to understand how the results of the audit procedures that they perform may
affect other aspects of the audit. The discussion provides an opportunity for more experienced engagement
team members, including the engagement partner, to share their insights based on their knowledge of the
entity, and for the team members to exchange information about the business risks.
All the team members should have a comprehensive knowledge of all aspects of the audit.
Ans.
All the team members should have a comprehensive knowledge of all aspects of the audit.

The assessment of the risks of material misstatement at the financial statement level is affected by the
auditor’s understanding of the control environment. Weaknesses in the control environment ordinarily will
lead the auditor to
Have more confidence in internal control and the reliability of audit evidence generated internally within the
entity.
Conduct some audit procedures at an interim date rather than at period end.
Modify the nature of audit procedures to obtain more persuasive audit evidence.
Decrease the number of locations to be included in the audit scope.
Ans.
Modify the nature of audit procedures to obtain more persuasive audit evidence.

The probability of an auditor’s procedures leading to the conclusion that a material error does not exist in an
account balance when, in fact, such error does exist referred to as
prevention risk. inherent risk control risk. detection risk.
Ans.
detection risk.

An auditor decides to increase the assessed level of control risk from that originally planned on the basis of
audit evidence gathered and evaluated. To achieve an overall audit risk level that is substantially the same as
the planned audit risk level, the auditor would
Decrease substantive testing.
Increase materiality levels.
Increase inherent risk.
Decrease detection risk.
Ans.
Decrease detection risk.

Which statement is incorrect regarding the extent of tests of controls?


The auditor designs tests of controls to obtain sufficient appropriate audit evidence that the controls
operated effectively throughout the period of reliance.
The more the auditor relies on the operating effectiveness of controls in the assessment of risk, the lesser is
the extent of the auditor’s tests of controls.
If the rate of expected deviation is expected to be too high, the auditor may determine that tests of controls
for a particular assertion may not be effective.
Because of the inherent consistency of IT processing, the auditor may not need to increase the extent of
testing of an automated control.
Ans.
The more the auditor relies on the operating effectiveness of controls in the assessment of risk, the lesser is
the extent of the auditor’s tests of controls.

Which type of risk does the management of a company have the most control over in the short term?
Inherent risk Control risk Detection risk Sufficiency risk
Ans.
Control risk
If the results of the auditor’s expert’s work do not provide sufficient appropriate audit evidence or are not
consistent with other audit evidence, the auditor should
Report the matter to the appropriate regulatory agency of the government.
Resolve the matter.
Withdraw from the engagement.
Express an unqualified opinion with reference to the work of the expert.
Ans.
Resolve the matter.

Which of the following is incorrect regarding PSA 315?


The purpose of this PSA is to establish standards and to provide guidance on obtaining an understanding of
the entity and its environment, including its internal control, and on assessing the risks of material
misstatement in a financial statement audit.
This PSA requires the auditor to make risk assessments at the financial statement and assertion levels based
on an appropriate understanding of the entity and its environment, including its internal control.
The requirements and guidance of this PSA are to be applied in conjunction with the requirements and
guidance provided in other PSAs.
This PSA discusses the auditor’s responsibility to determine overall responses and to design and perform
further audit procedures whose nature, timing, and extent are responsive to the risk assessments.
Ans.
This PSA discusses the auditor’s responsibility to determine overall responses and to design and perform
further audit procedures whose nature, timing, and extent are responsive to the risk assessments.

Opportunities to misappropriate assets increase when there are


Known or anticipated future employee layoffs.
Promotions, compensation, or other rewards inconsistent with expectations.
Recent or anticipated changes to employee compensation or benefit plans.
Inventory items that are small in size, of high value, or in high demand.
Ans.
Inventory items that are small in size, of high value, or in high demand.

The two types of intentional misstatements that are relevant to the auditor’s consideration of fraud include,
misstatements resulting from fraudulent financial reporting and misstatements resulting from
misappropriation of assets. Fraudulent financial reporting least likely involve.
Deception such as manipulation, falsification (including forgery), or alteration of accounting records or
supporting documents from which the financial statements are prepared
Misrepresentation in, or intentional omission from, the financial statements of events, transaction or other
significant information.
Intentional misapplication of accounting principles relating to measurement, recognition, classification,
presentation or disclosure
Embezzling receipts, stealing physical assets or intellectual property, causing an entity to pay for goods and
services not received, or using an entity’s assets for personal use.
Ans.
Embezzling receipts, stealing physical assets or intellectual property, causing an entity to pay for goods and
services not received, or using an entity’s assets for personal use.

On the basis of the audit evidence gathered and evaluated, an auditor decides to increase the assessed level of
control risk from that originally planned. To achieve an overall audit risk level that is substantially the same as
the planned audit risk level, the auditor would Decrease substantive testing
Decrease detection risk
Increase inherent risk
Increase materiality levels
Ans.
Decrease detection risk

After gaining an understanding of internal control and assessing the risks of material misstatement, an
auditor decided to perform tests of controls. The auditor most likely decided that
Additional evidence to support a further reduction in control risk is not available.
It is not possible or practicable to reduce the risks of material misstatement at the assertion level to an
acceptably low level with audit evidence obtained only from substantive test procedures.
There were many internal control weaknesses that could allow misstatements to enter the accounting
system.
An increase in the assessed level of control risk is justified for certain financial statement assertions.
Ans.
It is not possible or practicable to reduce the risks of material misstatement at the assertion level to an
acceptably low level with audit evidence obtained only from substantive test procedures.

Which statement is incorrect regarding the extent of further audit procedures?


Extent includes the quantity of a specific audit procedure to be performed
The extent of an audit procedure is determined by the judgment of the auditor after considering the
materiality, the assessed risk, and the degree of assurance the auditor plans to obtain.
The auditor ordinarily decreases the extent of audit procedures as the risk of material misstatement
increases.
Increasing the extent of an audit procedure is effective only if the audit procedure itself is relevant to the
specific risk.
Ans.
The auditor ordinarily decreases the extent of audit procedures as the risk of material misstatement
increases.

Inherent risk is ________related to detection risk and ________ related to the amount of audit evidence.
Directly, inversely Directly, directly Inversely, inversely Inversely, directly
Ans.
Inversely, directly

Which of the following statements is correct concerning an auditor’s assessment of control risk?
Evidence about the operation of internal control in prior audits may not be considered during the current
year’s assessment of control risk
The basis for an auditor’s conclusions about the assessed level of control risk need not be documented unless
control risk is assessed at the maximum level
The lower the assessed level of control risk, the less assurance the evidence must provide that the control
procedures are operating effectively
Assessing control risk may be performed concurrently during an audit with obtaining an understanding of
the entity’s internal control
Ans.
Assessing control risk may be performed concurrently during an audit with obtaining an understanding of the
entity’s internal control

Inquiries directed towards those charged with governance may most likely
Relate to their activities concerning the design and effectiveness of the entity’s internal control and whether
management has satisfactorily responded to any findings from these activities.
Help the auditor understand the environment in which the financial statements are prepared.
Relate to changes in the entity’s marketing strategies, sales trends, or contractual arrangements with its
customers.
Help the auditor in evaluating the appropriateness of the selection and application of certain accounting
policies.
Ans.
Help the auditor understand the environment in which the financial statements are prepared.

The audit risk model consists of: AR = IR x CR x DR

The detection risk is the dependent variable. What is the acceptable level of detection risk if the assessed
level of Inherent risk is High and the Control risk is Low?

Highest Medium Lower Higher


Ans.
Medium

Detection risk is
The risk that the auditor give an inappropriate audit opinion when the financial statements are materially
misstated
The risk that a misstatement, that could occur in an account balance or class of transactions and that could be
material individually or when aggregated with misstatements in other balances or classes, will not be
prevented or detected and corrected on a timely basis by the accounting and internal control systems
The risk that an auditor’s substantive procedures will not detect a misstatement that exists in an account
balance or class or transactions that could be material, individually or when aggregated with misstatements
in other balances or classes.
The susceptibility of an account balance or class of transactions to misstatement that could be material,
individually or when aggregated with misstatements in other balances of classes, assuming that there were no
related internal controls.
Ans.
The risk that an auditor’s substantive procedures will not detect a misstatement that exists in an account
balance or class or transactions that could be material, individually or when aggregated with misstatements
in other balances or classes.

The type of transactions that ordinarily have high inherent risk because they involve management judgment
or assumptions are referred to as
estimation transactions
non-routine transactions.
routine transactions.
related-party transactions.
Ans.
estimation transactions

Analytical procedures used during risk assessment in an audit should focus on


Reducing the scope of tests of controls and substantive tests
Providing assurance that potential material misstatements will be identified
Enhancing the auditor’s understanding of the client’s business
Assessing the adequacy of the available evidence
Ans.
Enhancing the auditor’s understanding of the client’s business

Which statement is incorrect regarding the nature of further audit procedures?


The nature of further audit procedures refers to their purpose and their type.
Certain audit procedures may be more appropriate for some assertions than others.
The higher the auditor’s assessment of risk, the less reliable and relevant is the audit evidence sought by the
auditor from substantive procedures.
The auditor is required to obtain audit evidence about the accuracy and completeness of information
produced by the entity’s information system when that information is used in performing audit procedures.
Ans.
The higher the auditor’s assessment of risk, the less reliable and relevant is the audit evidence sought by the
auditor from substantive procedures.

Which of the following is not component of audit risk?


Detection risk Business risk Control risk Inherent risk
Ans.
Business risk

PSA 315 requires

The auditor to obtain an understanding of the entity and its environment, including its internal control.

Discussion among the engagement team about the susceptibility of the entity’s financial statements to
material misstatement.

The auditor to identify and assess the risks of material misstatement at the financial statement and assertion
levels.

All of the above.


Ans.
All of the above.

The auditor’s substantive procedures should include the following audit procedures related to the financial
statement closing process:

I. Agreeing the financial statements to the underlying accounting records.


II. Examining material journal entries and other adjustments made during the course of preparing the
financial statements.

I and II
I only
II only
Neither I nor II
Ans.
I and II

A client maintains perpetual inventory records in both quantities and peso. If the assessed level of control risk
is high, an auditor would probably
Insist that the client perform physical counts of inventory items several times during the year

Apply gross profit tests to ascertain the reasonableness of the physical counts

Increase the extent of tests of controls of the inventory cycle

Request the client to schedule the physical inventory count at the end of the year
Ans.
Request the client to schedule the physical inventory count at the end of the year

When there are numerous property and equipment transactions during the year, an auditor who plans to
assess control risk at a low level usually performs

Tests of controls and extensive tests of property and equipment balances at the end of the year.

Analytical procedures for current year property and equipment transactions.

Tests of controls and limited tests of current year property and equipment transactions.

Analytical procedures for property and equipment balances at the end of the year.
Ans.
Tests of controls and limited tests of current year property and equipment transactions.

Which of the following conditions or events may create incentives/pressures to commit fraud?

Inadequate system of authorization and approval of transactions.

Lack of mandatory vacations for employees performing key control functions.

Excessive pressure on management or operating personnel to meet financial targets established by those
charged with governance, including sales or profitability incentive goals.

Inadequate access controls over automated records.


Ans.
Excessive pressure on management or operating personnel to meet financial targets established by those
charged with governance, including sales or profitability incentive goals.

Which of the following factors is not a good indicator of potential financial failure?

Client is constantly short of cash and working capital.

Client’s retained earnings were reduced by half as a result of a large dividend payout.

Client relies heavily on debt financing, especially by financing permanent assets with short-term loans

Client has had increasing net losses for several years.


Ans.
Client’s retained earnings were reduced by half as a result of a large dividend payout.

Which of the following is most likely to be presumed to represent a fraud risk on an audit?
Capitalization of repair and maintenance expense into the property, plant and equipment asset account

Improper revenue recognition

Improper interest expense accrual

Introduction of significant new products


Ans.
Improper revenue recognition

Which statement is incorrect regarding analytical procedures?

Analytical procedures may be helpful in identifying the existence of unusual transactions or events, and
amounts, ratios, and trends that might indicate matters that have financial statement and audit implications.

In performing analytical procedures as risk assessment procedures, the auditor develops expectations about
plausible relationships that are reasonably expected to exist.

When comparison of those expectations with recorded amounts or ratios developed from recorded amounts
yields unusual or unexpected relationships, the auditor considers those results in identifying risks of material
misstatement.

When such analytical procedures use data aggregated at a high level (which is often the situation), the results
of those analytical procedures provide a clear-cut indication about whether a material misstatement may
exist.
Ans.
When such analytical procedures use data aggregated at a high level (which is often the situation), the results
of those analytical procedures provide a clear-cut indication about whether a material misstatement may
exist.

Which of the following factors best define the materiality of audit risk?

1) Volume of transactions.
2) Degree of system integration.
3) Years since last audit.
4) Significant management turnover.
5) Value of assets at risk.
6) Average value per transaction.
7) Result of last audit.

1 through 7
2,4,and 7
1,5, and 6
3,4, and 6
Ans.
1,5, and 6

Your client, a merchandising concern, has annual sales of P30,000,000 and a 40% gross profit rate. Tests
reveal that 2% of the peso amount of purchases do not get into inventory because of breakage and inventory
pilferage by employees. The company estimates that these losses could be reduced to 0.5% of purchases by
designing and implementing certain controls costing approximately P350,000. Should the controls be
designed and implemented?

Yes, regardless of cost-benefit considerations, because the situation involves employee theft.

Yes, because the ideal system of internal control is the most extensive one.

No, because the cost of designing and implementing the added controls exceeds the projected savings.

Yes, because the expected benefits to be derived exceed the cost of the added controls.
Ans.
No, because the cost of designing and implementing the added controls exceeds the projected savings.

During the initial planning phase of an audit, a CPA most likely would

Identify specific internal control activities that are likely to prevent fraud

Evaluate the reasonableness of the client’s accounting estimates

Discuss the timing of the audit procedures with the client’s management

Inquire of the client’s attorney as to whether any unrecorded claims are probable of assertion
Ans.
Discuss the timing of the audit procedures with the client’s management

An auditor may decide to assess control risk at the maximum level for certain assertions because the auditor
believes

Controls are unlikely to pertain to the assertions.

The entity’s control components are interrelated.

Sufficient appropriate audit evidence to support the assertions is likely to be available.

More emphasis on tests of controls than substantive tests is warranted.


Ans.
Controls are unlikely to pertain to the assertions.

If the auditor sets the preliminary judgment about materiality at a relatively low peso amount,

more evidence will be required than for a high amount.


less evidence will be required than for a high amount
the same amount of evidence will be required as for a high peso amount.
the amount of evidence required will not be affected.
Ans.
more evidence will be required than for a high amount.

The auditor should determine overall responses to address the risks of material misstatement at the financial
statement level. Such responses least likely include

Emphasizing to the audit team the need to maintain professional skepticism in gathering and evaluating audit
evidence.
Assigning more experienced staff or those with special skills or using experts.

Incorporating additional elements of unpredictability in the selection of further audit procedures to be


performed.

Performing substantive procedures at an interim date instead of at period end.


Ans.
Performing substantive procedures at an interim date instead of at period end.

A potential business risk created by industry development may most likely include

Increased product liability


Increased legal exposure
The entity does not have the personnel or expertise to deal with the changes in the industry
Loss of financing due to entity’s inability to meet financing requirements.
Ans.
The entity does not have the personnel or expertise to deal with the changes in the industry

According to PSA 330 (The Auditor’s Procedures in Response to Assessed Risks), an auditor who plans to rely
on controls that have not changed since they were last tested should test the operating effectiveness of such
controls at least once every

Second audit
Third audit
Fourth audit
Fifth audit
Ans.
Third audit

Which statement is incorrect regarding the timing of tests of controls?

Audit evidence pertaining only to a point in time may be sufficient for the auditor’s purpose, for example,
when testing controls over the entity’s physical inventory counting at the period end.

If the auditor plans to rely on controls that have changed since they were last tested, the auditor should test
the operating effectiveness of such controls in the current audit.

If the auditor plans to rely on controls that have not changed since they were last tested, the auditor should
test the operating effectiveness of such controls at least once in every second audit.

When there are a number of controls for which the auditor determines that it is appropriate to use audit
evidence obtained in prior audits, the auditor should test the operating effectiveness of some controls each
audit.
Ans.
If the auditor plans to rely on controls that have not changed since they were last tested, the auditor should
test the operating effectiveness of such controls at least once in every second audit.

Which of the following auditing procedures most likely would assist an auditor in identifying related party
transactions?
Inspecting correspondence with lawyers for evidence of unreported contingent liabilities.

Vouching accounting records for recurring transactions recorded just after the balance sheet date.

Reviewing confirmations of loans receivable and payable for indications of guarantees.

Performing analytical procedures for indications of possible financial difficulties.


Ans.
Reviewing confirmations of loans receivable and payable for indications of guarantees.

Analytical procedures enable the auditor to predict the balance or quantity of an item under audit.
Information to develop this estimate can be obtained from all the following except

Comparison of financial data with data for comparable prior periods, anticipated results(e.g., budgets and
forecasts), and similar data for the industry in which the entity operates.

Study of the relationships of elements of financial data that would be expected to conform to a predictable
pattern based upon the entity’s experience

Study the relationships of financial data with relevant non-financial data

Tracing transactions through the system to determine whether procedures are being applied as prescribed
Ans.
Tracing transactions through the system to determine whether procedures are being applied as prescribed

Analytical procedures used planning an audit should focus on

Reducing the scope of tests of controls and substantive test.

Providing assurance that potential material misstatements will be identified

Enhancing the auditor’s understanding of the client’s business and identifying areas of potential risk.

Assessing the adequacy of the available evidential matter.


Ans.
Enhancing the auditor’s understanding of the client’s business and identifying areas of potential risk.

An auditor’s analytical procedures most likely would be facilitated if the entity

Segregates obsolete inventory before the physical inventory count

Uses a standard cost system that produces variance reports

Corrects material weaknesses in internal control before the beginning of the audit

Develops its data from sources solely within the entity


Ans.
Uses a standard cost system that produces variance reports

The auditor significant fluctuations in key elements of the company’s financial statements. If management is
unable to provide an acceptable explanation, the auditor should
Consider the matter a scope limitation
Perform additional audit procedures to investigate the matter further
Intensify the examination with the expectation of detecting management fraud
Withdraw from the engagement
Ans.
Perform additional audit procedures to investigate the matter further

The objective of performing analytical procedures in planning an audit is to identify the existence of
Unusual transactions and events
Illegal acts that went undetected because of internal control weaknesses
Related-party transactions
Recorded transactions that were not properly authorized
Ans.
Unusual transactions and events

Which of the following would not be considered an analytical procedure?

Estimating payroll expense by multiplying the number of employees by the average hourly wage rate and the
total hours by the average hourly wage

Projecting an error rate by comparing the results of a statistical sample with the actual population
characteristics.

Computing accounts receivable turnover by dividing credit sales by the average net receivables.

Developing the expected sales based on the sales trend of the prior five years.
Ans.
Projecting an error rate by comparing the results of a statistical sample with the actual population
characteristics.

Analytical procedures used in the overall review stage of the audit generally include

Retesting controls that appeared to be ineffective during the assessment of control risk.

Considering unusual or unexpected account balances that were not previously identified.

Gathering evidence concerning account balances that have not changed from the prior year.

Performing tests of transactions to corroborate management’s financial statement assertions.


Ans.
Considering unusual or unexpected account balances that were not previously identified.

Analytical procedures used in planning an audit should focus on

Reducing the scope of tests of controls and substantive tests


Providing assurance that potential material misstatements will be identified
Enhancing the auditor’s understanding of the client’s business
Assessing the adequacy of the available evidence
Ans.
Enhancing the auditor’s understanding of the client’s business
For all audits of financial statements made in accordance with PSAs, the use of analytical procedures is
important
1) In assessment of risk of material misstatement
2) as substantive audit procedure
3) as an overall review at the completion stage
1) Yes 2) No 3) Yes
1) No 2) Yes 3) No
1) No 2) Yes 3) Yes
1) Yes 2) No 3) No
Ans.
1) Yes 2) No 3) Yes

Which of the following statements is correct with respect to the auditor’s use of analytical procedures?

Analytical procedures are time-saving procedures that auditors may employ at their discretion.

Analytical procedures are powerful tools that are required to be used during the planning and testing phases
of the audit.

Analytical procedure may be used to identify misstatements in a client’s accounts

Analytical procedures are required to be used during the planning and completion phase of the audit.
Ans.
Analytical procedures are required to be used during the planning and completion phase of the audit.

Analytical procedures performed in the overall review stage of an audit suggest that several accounts have
unexpected relationships. The results of these procedures most likely indicate that

The communication with the audit committee should be revised.


Irregularities exist among the relevant account balances.
Additional substantive tests of details are required.
Internal control activities are not operating effectively.
Ans.
Additional substantive tests of details are required.

Evaluations of financial information made by a study of plausible relationships among both financial and non-
financial data. It also encompasses the investigation of identified fluctuations and relationships that are
inconsistent with other relevant information or that differ form expected values by a significant amount.

Audit planning
Audit evidence
Analytical procedures
Inspection
Ans.
Analytical procedures

Analytical procedures are performed in the following order:


Calculate predictions and compare them to the recorded amount; define a significant difference; develop an
expectation; investigate significant differences.

Calculate predictions and compare them to the recorded amount; investigate significant differences; define a
significant difference; develop an expectation

Develop an expectation; define a significant difference; calculate predictions and compare them to the
recorded amount; investigate significant differences.

Develop an expectation; calculate predictions and compare them to them recorded amount; define a
significant difference; investigate significant differences.
Ans.
Develop an expectation; calculate predictions and compare them to them recorded amount; define a
significant difference; investigate significant differences.

Which of the following most likely would indicate the existence of related parties?

Writing down obsolete inventory just before year-end.


Failing to correct previously identified internal control deficiencies.
Depending on a single product for the success of the entity.
Borrowing money at an interest rate significantly below the market rate.
Ans.
Borrowing money at an interest rate significantly below the market rate.

Analytical procedures performed in the planning stage of an audit suggest that several accounts have
unexpected relationships. The results of these procedures most likely would indicate that:

Irregularities exist among the relevant account balances.


Additional test of details are required.
Internal control activities are not operating effectively.
The communication with the audit committee should be revised.
Ans.
Additional test of details are required.

Which of the following results from analytical procedure might indicate obsolete inventory?

A decline in inventory turnover


A decline in days sales in inventory
A decline in the gross margin ratio
An increase in operating margin
Ans.
A decline in inventory turnover

If, when performing analytical procedures, an auditor observes that operating income has declined
significantly between the preceding year and the current year, the auditor should next

require that the decline be disclosed in the financial statements.

consider the possibility that the financial statements may be materially misstated

inform management that a qualified opinion on the financial statement will be necessary.
determine management’s responsibility for the decline and discuss the issue with the audit committee.
Ans.
consider the possibility that the financial statements may be materially misstated

When must an auditor perform analytical review procedures in a financial statement audit?

Testing controls over financial cycles


Performing test to substantiate balances
Planning the nature, timing and extent of procedures
Performing tests to substantiate transactions
Ans.
Planning the nature, timing and extent of procedures

Auditors try to identify predictable relations when using analytical procedures. Relationships involving
transactions from which of the following accounts most likely would yield the highest level of evidence?

Accounts payable
Accounts receivable
Payroll expense
Advertising expense
Ans.
Payroll expense

Analytical procedures are used in an audit because it is assumed of financial statements that

Management fraud can be discovered using such procedures.

It is plausible that no relationship among data exists.

Analytical procedures are used as tests of controls

Plausible relationships among data may reasonably be expected to exist and continue in the absence of known
conditions to the contrary.
Ans.
Plausible relationships among data may reasonably be expected to exist and continue in the absence of known
conditions to the contrary.

Which of the following represents a procedure that the auditor may use because plausible relationships
among financial statement balances are expected to exist?

Attributes testing
Enterprise risk assessment
Inherent tests of control
Analytical review
Ans.
Analytical review

Which of the following nonfinancial information would an auditor most likely consider in performing
analytical procedures during the planning phase of an audit?
Turnover of personnel in the accounting department
Objectivity of audit committee members
Square footage of selling space
Management’s plans to repurchase stock
Ans.
Square footage of selling space

Which of the following is not a reason for utilizing analytical procedures

To assess the entity’s ability to continue as a going- concern

To identify areas with no unusual fluctuations so that fewer detailed tests may be performed on those
accounts

To determine the magnitude of error in the FS

To highlight changes from the prior year to the current year so that trends can be identified which influence
audit planning.
Ans.
To determine the magnitude of error in the FS

In performing an audit , which one of the following procedures would be considered an analytical procedure?

Comparing last year’s interest expense with this year’s interest expense
Comparing signatures on checks with the signatures of authorized check signers
Reviewing initials on received documents
Reviewing procedures followed in receiving, depositing, and disbursing cash
Ans.
Comparing last year’s interest expense with this year’s interest expense

With respect to the auditor’s planning of a year-end examination which of the following statements is always
true?
An engagement should not be accepted after the fiscal year end.
An inventory count must be observed at the balance sheet date
The client’s committee should not be told of the specific audit procedures that will be performed.
It is an acceptable practice to carry out substantial parts of the examination at interim dates.
Ans.
It is an acceptable practice to carry out substantial parts of the examination at interim dates.

Which statement best describe the emphasis of the systems and substantive approaches in the audit plan?
The system approach focuses on testing controls to make sure they are effective, while the substantive
approach is the detailed testing of specific accounts for accuracy.
The system approach focuses on detailed testing of specific accounts for accuracy, while the substantive
approach is the testing controls to make sure they are effective.
The systems approach focuses on the use of computer systems to aid in the audit while the substantive
approach focuses on more manual tests.
A thoroughly designed systems approach to auditing can eliminate the need for any substantive procedures.
Ans.
The system approach focuses on testing controls to make sure they are effective, while the substantive
approach is the detailed testing of specific accounts for accuracy.
Audit program is basically s listing of all the things the auditor will do to gather sufficient, competent
evidence. Which item would not be contained in an audit program?
Staff assigned to the audit
List of specific procedures (tasks) to be performed and the objectives to be met that relate primarily to
financial statements assertions.
Documentation of system being reviewed
Estimated time required to perform each task.
Ans.
Documentation of system being reviewed

An audit plan is a
detailed plan of analytical procedure and all substantive test to be performed in the course of the audit.
document that provides an overview of the company and a general plan for the audit work to be
accomplished, timing of the work, and other matters of concern to the audit.
generic document that auditing firms have developed to lead the process of the audit through a systematic
and logical process.
budget of the time that should be necessary to complete each phase of the audit procedures.
Ans.
document that provides an overview of the company and a general plan for the audit work to be
accomplished, timing of the work, and other matters of concern to the audit.

B&C, auditors, have been accepted as the auditors of City Corporation. What are B&C’s responsibilities with
regard to contacting City Corporation’s predecessor auditors?
If City Corporation had a disagreement with predecessor auditors, R&O should not contact the predecessor
auditors.
B&C should attempt communication with the predecessor auditors under any circumstances.
B&C should attempt communications with the predecessor auditors and ask if they had any accounting policy
disagreements with City Corporation.
It would be unethical for B&C to ask the predecessor auditors about the integrity of City Corporation’s
management.
Ans.
B&C should attempt communications with the predecessor auditors and ask if they had any accounting policy
disagreements with City Corporation.

An audit program provides proof that


sufficient competent evidential matter was obtained.
the work was adequately planned.
there was compliance with generally accepted standards of reporting
there was a proper study and evaluation internal control.
Ans.
the work was adequately planned.

The audit plan includes:


A description of the nature, timing and extent of planned risk assessment procedures sufficient to assess the
risk of material misstatement.
A description of the nature, timing and extent of planned further audit procedures at the assertion level for
each material class of transactions, account balances and disclosures. Other planned audit procedures that are
required to be carried out so that the engagement complies with our audit approach.
All of the above
Ans.
All of the above

In deciding whether to use the work of internal auditors, external auditors must evaluate the internal
auditors’
objectivity and competence
independence and professionalism
education and certification
age and gender
Ans.
objectivity and competence

Which of the following is not a component of audit planning?


Observing the client’s annual physical inventory taking and making test counts of selected items
Making arrangements with the client concerning the timing of audit fieldwork and use of the client’s staff in
completing certain phases of the examination
Obtaining an understanding of the business
Developing audit programs
Ans.
Observing the client’s annual physical inventory taking and making test counts of selected items

The element of the audit planning process most likely to be agreed upon with the client before
implementation of the audit strategy is the determination of the
methods of statistical sampling to be used in confirming accounts receivable.
pending legal matters to be included in the inquiry of the client’s attorney.
evidence to be gathered to provide a sufficient basis for the auditor’s opinion.
schedules and analyses to be prepared by the client’s staff.
Ans.
schedules and analyses to be prepared by the client’s staff.

An audit program is
the detailed plan of audit procedures to be performed in the course of the audit.
an overview of the company and a general plan for the audit work to be accomplished. a generic document
that auditing firms have developed to lead the process of the audit through a systematic and logical process.
a budget of the time that should be necessary to complete each phase of the audit procedures.
Ans.
the detailed plan of audit procedures to be performed in the course of the audit.

How can the audit program best be described at the beginning of the audit process?
Temporary Conclusive Confirmed Optional
Ans.
Temporary

An initial (first-time) audit requires more audit time to complete than a recurring audit. One of the reason for
this is that
new auditors are usually assigned to an initial audit.
predecessor auditors need to be consulted
the client’s business, industry and internal control are unfamiliar to the auditor and need to be carefully
studied.
a larger proportion of customer accounts receivable need to be confirmed on an initial audit.
Ans.
the client’s business, industry and internal control are unfamiliar to the auditor and need to be carefully
studied.

As a part of audit planning, CPAs should design audit programs for each individual audit and should include
audit steps and procedures to
detect and eliminate fraud.
increase the amount of management information available.
provide assurances that the objectives of the audit are met.
ensure that only material items are audited.
Ans.
provide assurances that the objectives of the audit are met.

After an auditor has been engaged to perform the first audit for a nonpublic entity, the client requested to
change the engagement to a review. In which of the following situations would there be a reasonable basis to
comply with the client’s request?

The client’s bank required an audit before committing to a loan, but the client subsequently acquired
alternative financing.

The auditor was prohibited by the client from corresponding with the client’s legal counsel.

Management refused to sign the client representation letter.

The auditing procedures were substantially complete and the auditor determined that an unqualified opinion
was warranted, but there was a disagreement concerning the audit fee.
Ans.
The client’s bank required an audit before committing to a loan, but the client subsequently acquired
alternative financing.

Which one of the following is not considered a valid source of information about the client’s processes?

Confirmation of third parties


Review of the client’s budget
A tour of the client’s plant
Management inquiry
Ans.
Confirmation of third parties

Which of the following is not true regarding planning in an electronic environment?

The definition of auditing is not changed


The purposes of auditing is not changed
The procedures used are not changed
Auditing standards are not changed
Ans.
The procedures used are not changed
When planning the audit, if the auditor has no reason to believe that illegal acts exist, the auditor should

make inquiries of management regarding their policies and their knowledge of violations, and then rely on
normal audit procedures to detect errors, irregularities, and illegalities.

still include some audit procedures designed specifically to uncover illegalities.

ignore the topic.

include audit procedures which have a strong probability of detecting illegal acts.
Ans.
make inquiries of management regarding their policies and their knowledge of violations, and then rely on
normal audit procedures to detect errors, irregularities, and illegalities.

As part of audit planning, CPAs should design audit programs for each individual audit and should include
audit steps and procedures to

Detect and eliminate fraud


Increase the amount of management information available
Provide assurances that the objectives of the audit are met
Ensure that only material items are audited
Ans.
Provide assurances that the objectives of the audit are met

Auditing standards require a successor auditor to communicate with the predecessor auditor. The Code of
Ethics requires confidentiality; therefore, the client’s permission must be obtained before the communication
can be made by

the successor auditor.

the predecessor auditor.

both the successor and predecessor auditor

neither, since this is one of the exceptions to confidentiality in the Code of Professional Conduct.
Ans.
the successor auditor.

The elements of the audit planning process most likely to be agreed upon with the client before
implementation of the audit strategy is the determination of the

evidence to be gathered to provide a sufficient basis for the auditor’s opinion.

procedures to be undertaken to discover litigation, claims, and assessments.

timing of inventory observation procedures to be performed.

pending legal matters to be included in the inquiry of the client’s attorney.


Ans.
timing of inventory observation procedures to be performed.
When a CPA is approached to perform an audit for the first time, the CPA should make inquiries of the
predecessor auditor. This is a necessary procedure because the predecessor may be able to provide the
successor with information that will assist the successor in determining

whether the predecessor’s work should be utilized.


whether the company follows the policy of rotating its auditors.
whether, in the predecessor’s opinion, internal control of the company has been satisfactory.
whether the engagement should be accepted.
Ans.
whether the engagement should be accepted.

In general, a material weakness in internal control may be defined as a condition in which material errors or
irregularities may occur and not be detected within a timely period by

An independent auditor during tests of controls.


Management when reviewing interim financial statements and reconciling account balances.
Employees in the normal course of performing their assigned functions.
Outside consultants who issue a special-purpose report on internal control structure.
Ans.
Employees in the normal course of performing their assigned functions.

Corporate directors, management, external auditors, and internal auditors all play important roles in creating
a proper control environment. Top management is primarily responsible for

Establishing a proper environment and specifying overall internal control.

Reviewing the reliability and integrity of financial information and the means used to collect and report such
information.

Ensuring that external and internal auditors adequately monitor the control environment.

Implementing and monitoring controls designed by the board of directors.


Ans.
Establishing a proper environment and specifying overall internal control.

The primary responsibility for designing, implementing and maintaining internal control, and the tone of
internal control typically originates, rests with

Internal auditors
The CFOT
he external auditor
The management/TCWG
Ans.
The management/TCWG

Proper segregation of functional responsibilities calls for separation of the functions of

Authorization, execution, and payment.


Authorization, recording, and custody.
Custody, execution, and reporting.
Authorization, payment, and recording.
Ans.
Authorization, recording, and custody.

In an audit of financial statements, an auditor’s primary consideration regarding a control is whether it

Reflects management’s philosophy and operating style.


Affects management’s financial statement assertions.
Provides adequate safeguards over access to assets.
Enhances management’s decision-making processes.
Ans.
Affects management’s financial statement assertions.

Internal control procedures are not designed to provide reasonable assurance that

Transactions are executed in accordance with management's authorization.

Access to assets is permitted only in accordance with management's authorization.

Irregularities will be eliminated.

The recorded accountability for assets is compared with the existing assets at reasonable intervals.
Ans.
Irregularities will be eliminated.

Effective internal control

Eliminates risk and potential loss to the organization.

Cannot be circumvented by management.

Is unaffected by changing circumstances and conditions encountered by the organization.

Reduces the need for management to review exception reports on a day-to-day basis.
Ans.
Reduces the need for management to review exception reports on a day-to-day basis.

When considering the effectiveness of a system of internal accounting control, the auditor should recognize
that inherent limitations do exist. Which of the following is an example of an inherent limitation in a system of
internal accounting control?
The effectiveness of procedures depends on the segregation of employee duties.

Procedures are designed to assure the execution and recording of transactions in accordance with
management’s authorization.

In the performance of most control procedures, there are possibilities of errors arising from mistakes in
judgment

Procedures for handling large numbers of transactions are processed by electronic data processing
equipment
Ans.
In the performance of most control procedures, there are possibilities of errors arising from mistakes in
judgment

When considering internal control, an auditor must be aware of the concept of reasonable assurance, which
recognizes that

Employment of competent personnel provides assurance that the objectives of internal control will be
achieved

Establishment and maintenance of internal control is an important responsibility of the management and not
of the auditor

Cost of internal control procedures should not exceed the benefits expected to be derived from the control

Segregation of incompatible functions is necessary to ascertain that the control procedures effective.
Ans.
Cost of internal control procedures should not exceed the benefits expected to be derived from the control

According to PSA 400, which of the following is correct regarding internal control system?

Internal control system refers to all the policies and procedures adopted by the auditor to assist in achieving
management’s objective.

A strong environment, by itself, ensure the effectiveness of the internal control system.

In the audit of financial statements, the auditor is only concerned with those policies and procedures within
the accounting and internal control systems that are relevant to the financial statements.

The internal control system is confined to those matters which relate directly to the functions of the
accounting system.
Ans.
In the audit of financial statements, the auditor is only concerned with those policies and procedures within
the accounting and internal control systems that are relevant to the financial statements.

As generally conceived, the “audit committee” of a publicly held company should be made up of

Members of the board of directors who are not officers or employees.


Representatives of the major equity interests (bonds, preferred shares, ordinary shares).
The audit partner, the chief financial officer, the legal counsel, and at least one outsider.
Representatives from the client’s management, investors, suppliers, and customers.
Ans.
Members of the board of directors who are not officers or employees.

Which of the following most likely would not be considered an inherent limitation of the potential
effectiveness of an entity’s internal control?

Incompatible duties.
Mistakes in judgment.
Management override.
Collusion among employees.
Ans.
Incompatible duties.

The process designed and affected by those charged with governance, management, and other personnel to
provide reasonable assurance about the achievement of the entity’s objectives with regard to reliability of
financial reporting, effectiveness and efficiency of operations and compliance with applicable laws and
regulations.

Internal control
Accounting control
Administrative control
Control environment
Ans.
Internal control

Which of the following least likely affects the nature, timing, and extent of the procedures performed by the
auditor to obtain an understanding of the accounting and internal control systems of an audit client?

Materiality considerations
The auditor’s assessment of inherent risk
The level of acceptable detection risk
The size and complexity of the entity and of its control systems.
Ans.
The level of acceptable detection risk

When considering internal control, an auditor should be aware of the concept of reasonable assurance, which
recognizes that

Internal control may be ineffective due to mistakes in judgment and personal carelessness.

Adequate safeguards over access to assets and records should permit an entity to maintain proper
accountability.

Establishing and maintaining internal control is an important responsibility of management.

The cost of an entity’s internal control should not exceed the benefits expected to be derived.
Ans.
The cost of an entity’s internal control should not exceed the benefits expected to be derived.

Which of the following is a responsibility that should not be assigned to only one employee?

Access to securities in the company’s safe deposit box.


Custodianship of the cash working fund.
Reconciliation of bank statement.
Custodianship of tools and small equipment.
Ans.
Access to securities in the company’s safe deposit box.

Which of the following best describes the interrelated components of internal control?
Organizational structure, management, philosophy, and planning
Control environment, risk assessment, control activities, information and communication systems, and
monitoring

Risk assessment, backup facilities, responsibility accounting, and natural laws

Internal audit and management’s philosophy and operating style.


Ans.
Control environment, risk assessment, control activities, information and communication systems, and
monitoring

An entity’s ongoing monitoring activities often include

Periodic audits by the audit committee.


Reviewing the purchasing function.
The audit of the annual financial statements.
Control risk assessment in conjunction with quarterly reviews.
Ans.
Reviewing the purchasing function.

The overall attitude and awareness of an entity’s board of directors concerning the importance of internal
control usually is reflected in its

Computer-based controls.
Control environment.
System of segregation of duties.
Safeguards over access to assets.
Ans.
Control environment.

Which of the following is a responsibility that should not be assigned to only one employee?

Access to securities in the company’s safe deposit box.


Custodianship of the cash working fund.
Reconciliation of bank statement.
Custodianship of tools and small equipment.
Ans.
Access to securities in the company’s safe deposit box.

Control environment component of internal control

Consists of the policies and procedures that help ensure that management directives are carried out.

Includes the governance and management functions and the attitudes, awareness, and actions of those
charged with governance and management concerning the entity’s internal control and its importance in the
entity.

Is the entity process for identifying business risks relevant to financial reporting objectives and deciding
about actions to address those risks, and the results thereof?

Consists of the procedures and records established to initiate, record, process, and report entity transactions
(as well as events and conditions) and to maintain accountability for the related assets, liabilities, and equity.
Ans.
Includes the governance and management functions and the attitudes, awareness, and actions of those
charged with governance and management concerning the entity’s internal control and its importance in the
entity.

Proper segregation of functional responsibilities calls for separation of the functions of

Authorization, execution, and recording.


Custody, execution, and reporting.
Authorization, execution, and payment.
Authorization, payment, and recording.
Ans.
Authorization, execution, and recording.

Which of the following is not part of the control environment?

Management philosophy and operating style.


Organizational structure and methods of assigning authority and responsibility.
Information and communication systems.
The function of the board of directors and its committees.
Ans.
Information and communication systems.

Which of the following is not an element of a system of quality control?

Leadership responsibilities for quality within the firm


Acceptance and continuance of client relationships and specific engagements
Engagement performance
Measurement and review of performance
Ans.
Measurement and review of performance

Transaction authorization within an organization may be either specific or general. An example of specific
transaction authorization is the

Approval of a construction budget for a new warehouse


Setting of automatic reorder points
Establishment of a customer’s credit limits
Establishment of sales prices
Ans.
Approval of a construction budget for a new warehouse

Which of the following best describe the interrelated components of internal control?

Organizational structure, management philosophy, and planning.

Control environment, risk assessment, control activities, information and communication systems, and
monitoring.

Risk assessment, backup facilities, responsibility accounting and natural laws.


Legal environment of the firm, management philosophy, and organizational structure.
Ans.
Control environment, risk assessment, control activities, information and communication systems, and
monitoring.

A secondary purpose of the auditor's consideration of internal control is to provide

A basis for assessing control risk.

An assurance that the records and documents have been maintained in accordance with existing company
policies and procedures.

A basis for constructive suggestions about improvements in internal control structure.

A basis for the determination of the resultant extent of the tests to which auditing procedures are to be
restricted.
Ans.
A basis for constructive suggestions about improvements in internal control structure.

When obtaining an understanding of an entity’s control environment, an auditor should concentrate on the
substance of management’s policies and procedures rather than their form because

The auditor may believe that the policies and procedures are inappropriate for that particular entity

The board of directors may not be aware of management’s attitude toward the control environment

Management may establish appropriate policies and procedures but not act on them

The policies and procedures may be so ineffective that the auditor may assess control risk at the maximum
level
Ans.
Management may establish appropriate policies and procedures but not act on them

Which of the following activities would be an example of Physical Control?

Access to computer facilities and records is limited to authorized personnel.


Training programs are conducted to develop competence of newly hired personnel
Control and subsidiary accounts are reconciled on a regularly scheduled basis
Blank stock of all purchase orders and sales invoices are prenumbered
Ans.
Access to computer facilities and records is limited to authorized personnel.

Which of the following is not a component of an entity’s internal control?

Control risk
Monitoring
Control activities
Control environment
Ans.
Control risk
Which of the following does not relate to the firm’s quality control policies and procedures on engagement
performance?

Independence
Consultation
Direction
Supervision
Ans.
Independence

The policies and procedures that help ensure that management directives are carried out are referred to as
the:
Control environment
Control activities
Monitoring of controls
Information system
Ans.
Control activities

After considering a client’s internal control, an auditor has concluded that the system is well designed and is
functioning as anticipated. Under these circumstances, the auditor would most likely

Cease to perform further substantive tests


Not increase the extent of planned substantive tests
Increase the extent of anticipated analytical procedures
Perform all tests of controls to the extent outlined in the preplanned audit program
Ans.
Not increase the extent of planned substantive tests

Before relying on the system of internal control, the auditor obtains a reasonable degree of assurance that the
internal control procedures are in use and operating as planned. The auditor obtains this assurance by
performing planned

Substantive tests
Tests of controls
Transaction tests
Tests of trends and ratios
Ans.
Tests of controls

After considering internal control, an auditor might decide to

Increase the extent of tests of controls and substantive tests in areas where internal control is strong

Increase the extent of substantive tests in areas where internal control is weak

Reduce the extent of tests of controls in areas where internal control is strong

Reduce the extent of both substantive tests and tests of controls in areas where internal control is strong
Ans.
Increase the extent of substantive tests in areas where internal control is weak

Which statement is correct regarding the sufficiency and appropriateness of audit evidence?

Sufficiency is the measure of the quality of audit evidence.

Appropriateness is the measure of the quantity of audit evidence; that is, its relevance and its reliability in
providing support for, or detecting misstatements in, the classes of transactions, account balances, and
disclosures and related assertions.

The quantity of audit evidence needed is affected by the risk of misstatement (the greater the risk, the more
audit evidence is likely to be required) and also by the quality of such audit evidence (the higher the quality,
the less may be required).

Merely obtaining more audit evidence may compensate for its poor quality.
Ans.
The quantity of audit evidence needed is affected by the risk of misstatement (the greater the risk, the more
audit evidence is likely to be required) and also by the quality of such audit evidence (the higher the quality,
the less may be required).

Which of the following statements is incorrect regarding relevance of audit evidence?

A given set of audit procedures may provide audit evidence that is relevant to certain assertions, but not
others.

The auditor often obtains audit evidence from different sources or of a different nature that is relevant to the
same assertion.

Obtaining audit evidence relating to a particular assertion is a substitute for obtaining audit evidence
regarding another assertion.

None of the above.


Ans.
Obtaining audit evidence relating to a particular assertion is a substitute for obtaining audit evidence
regarding another assertion.

The auditor is examining copies of sales invoices only for the initials of the person responsible for checking
the extensions. This is an example of a

Test of controls
Substantive test
Dual purpose test
Test of balances
Ans.
Test of controls

An auditor wishes to perform tests of controls on a client’s cash disbursements procedures. If the controls
leave no audit trail of documentary evidence, the auditor most likely will test the procedures by

Confirmation and observation.


Observation and inquiry.
Analytical procedures and confirmation.
Inquiry and analytical procedures
Ans.
Observation and inquiry.

Confirmation is most likely to be a relevant form of evidence with regard to assertions about accounts
receivable when the auditor has concerns about the Receivables

Valuation
Classification
Existence
Completeness
Ans.
Existence

Each of the following might, by itself, form a valid basis for an auditor to decide to omit a test except for the

Difficulty and expense involved in testing a particular item.


Assessment of control risk at a low level.
Inherent risk involved.
Relationship between the cost of obtaining evidence and its usefulness.
Ans.
Difficulty and expense involved in testing a particular item.

Which of the following statements is correct concerning the use of negative confirmation requests?

Unreturned negative confirmation requests rarely provide significant explicit evidence.

Negative confirmation requests are effective when detection risk is low.

Unreturned negative confirmation requests indicate that alternative procedures are necessary.

Negative confirmation requests are effective when understatements of account balances are suspected.
Ans.
Unreturned negative confirmation requests rarely provide significant explicit evidence.

Observation

Consists of looking at a process or procedure being performed by others.

Consists of seeking information of knowledgeable persons, both financial and non-financial, throughout the
entity or outside the entity.

Is the process of obtaining a representation of information or of an existing condition directly from a third
party.

Is the auditor’s independent execution of procedures or controls that were originally performed as part of the
entity’s internal control.
Ans.
Consists of looking at a process or procedure being performed by others.
In which of the following circumstances would the use of the negative form of accounts receivable
confirmation most likely be justified?

A substantial number of accounts may be in dispute and the accounts receivable balance arises from sales to a
few major customers.

A substantial number of accounts may be in dispute and the accounts receivable balance arises from sales to
many customers with small balances.

A small number of accounts may be in dispute and the accounts receivable balance arises from sales to a few
major customers.

A small number of accounts may be in dispute and the accounts receivable balance arises from sales to many
customers with small balances.
Ans.
A small number of accounts may be in dispute and the accounts receivable balance arises from sales to many
customers with small balances.

The principal reason for an independent auditor to gather and evaluate audit evidence is to

Detect fraud.
Evaluate management.
Evaluate internal control.
Form an opinion of the financial statements
Ans.
Form an opinion of the financial statements

Which of the following should be considered by the auditor in deciding which means (or combination of
means) to use in selecting items for testing?

I. The risk of material misstatement related to the assertion being tested.


II. Audit efficiency.

I only
II only
Both I and II
Neither I nor II
Ans.
Both I and II

Which of the following generalizations in assessing the reliability of audit evidence is incorrect?

Audit evidence is more reliable when it is obtained from independent sources outside the entity.

Audit evidence that is generated internally is not affected by the effectiveness of the controls imposed by the
entity.

Audit evidence obtained directly by the auditor is more reliable than audit evidence obtained indirectly or by
inference.

Audit evidence is more reliable when it exists in documentary form.


Ans.
Audit evidence that is generated internally is not affected by the effectiveness of the controls imposed by the
entity.

Holding other planning considerations equal, a decrease in the amount of misstatement in a class of
transactions that an auditor could tolerate most likely would cause the auditor to

Apply the planned substantive tests prior to the balance sheet date

Perform the planned auditing procedures closer to the balance sheet date

Increase the assessed level of control risk for relevant financial statement assertions

Decrease the extent of auditing procedures to be applied to the class of transactions


Ans.
Perform the planned auditing procedures closer to the balance sheet date

Leadership responsibilities within the firm shall be assumed by the

Chief executive officer


Junior partners
Human resource personnel
Board of Accountancy
Ans.
Chief executive officer

Which of the following terms is used in the standard to describe the effects on the financial statements of
misstatements or the possible effects on the financial statements, if any, that are undetected due to an
inability to obtain sufficient appropriate audit evidence?

Persuasive
Pervasive
Material
Extensive
Ans.
Pervasive

The principal reason for an independent auditor to gather and evaluate audit evidence is to

Detect fraud.
Evaluate management.
Evaluate internal control.
Form an opinion of the financial statements
Ans.
Form an opinion of the financial statements

Which of the following types of audit evidence is the most persuasive?

Which of the following types of audit evidence is the most persuasive?


Client worksheets supporting cost allocations.
Bank statements obtained from the client.
Client representation letter.
Ans.
Bank statements obtained from the client.

Which of the following statements concerning evidential matter is correct?


Competent evidence supporting management's assertions should be convincing rather than merely
persuasive.

An effective internal control structure contributes little to the reliability of the evidence created within the
entity.

The cost of obtaining evidence is not an important consideration to an auditor in deciding what evidence
should be obtained.

A client's accounting data cannot be considered sufficient audit evidence to support the financial statements.
Ans.
A client's accounting data cannot be considered sufficient audit evidence to support the financial statements.

Accounting records least likely include


The records of initial entries and supporting records.
The general and subsidiary ledgers.
Work sheets and spreadsheets supporting cost allocations.
Comparable data about competitors (benchmarking).
Ans.
Comparable data about competitors (benchmarking).

Audit information is usually considered relevant when it is

Derived through valid statistical sampling.


Objective and unbiased.
Factual, adequate, and convincing.
Consistent with the audit objectives.
Ans.
Consistent with the audit objectives.

Which of the following generalizations does not relate to the appropriateness of evidence?

Audit evidence from external sources (for example, confirmation received from a third party) is more reliable
than that generated internally.

An auditor’s opinion, to be economically useful, is formed within reasonable time and based on evidence
obtained at a reasonable cost.

Audit evidence generated internally is more reliable when the related accounting and internal control systems
are effective.

Audit evidence obtained directly by the auditor is more reliable than that obtained from the entity.
Ans.
An auditor’s opinion, to be economically useful, is formed within reasonable time and based on evidence
obtained at a reasonable cost.
Which statement is incorrect regarding audit evidence?

Audit evidence is all the information used by the auditor in arriving at the conclusions on which the audit
opinion is based.

Audit evidence includes the information contained in the accounting records underlying the financial
statements and other information.

Audit evidence is cumulative in nature.

Auditors are expected to address all information that may exist.


Ans.
Auditors are expected to address all information that may exist.

Which of the following is an audit procedure that an auditor most likely would perform concerning litigation,
claims, and assessments?

Request the client’s lawyer to evaluate whether the client’s pending litigation, claims, and assessments
indicate a going concern problem.

Examine the legal documents in the client’s lawyer’s possession concerning litigation, claims, and
assessments to which the lawyer has devoted substantive attention.

Discuss with management its policies and procedures adopted for evaluating and accounting for litigation,
claims, and assessments.

Confirm directly with the client’s lawyer that all litigation, claims, and assessments have been recorded or
disclosed in the financial statements.
Ans.
Discuss with management its policies and procedures adopted for evaluating and accounting for litigation,
claims, and assessments.

The auditor should adopt one or a combination of the following approaches in the audit of an accounting
estimate:
I. Review and test the process used by management to develop the estimate.
II. Use an independent estimate for comparison with that prepared by management.
III. Review subsequent events which confirm the estimate made.

Any of the above


None of the above
Either I or II
I only
Ans.
Any of the above

It means an approximation of the amount of an item in the absence of a precise means of measurement

Accounting estimate
Accounting policy
Accounting error
Accounting change
Ans.
Accounting estimate

It is the responsibility of the auditor to evaluate the reasonableness of the accounting estimates made by
management. Which one of the following approaches would the auditor not use when evaluating the
reasonableness of the estimate?

Review and test management’s process to develop the estimate.


Calculate an independent expectation of the estimate.
Confirm the estimate with independent parties.
Review subsequent events or transactions occurring prior to completion of fieldwork.
Ans.
Confirm the estimate with independent parties.

Management prepares accounting estimates and the auditor is responsible for evaluating the reasonableness
of the estimates. Which of the following would not be an auditor’s objective when evaluating estimates?

All accounting estimates which could be material to the financial statements have been developed.

The accounting estimates developed by management are accurate with 100% certainty.

The accounting estimates developed by management are reasonable.

The accounting estimates are presented in accordance with PFRS.


Ans.
The accounting estimates developed by management are accurate with 100% certainty.

In evaluating the assumptions on which the estimate is based, the auditor would need to pay particular
attention to assumptions which are

Reasonable in light of actual results in prior periods.


Consistent with those used for other accounting estimates.
Consistent with management’s plans which appear appropriate.
Subjective or susceptible to material misstatement.
Ans.
Subjective or susceptible to material misstatement.

Which of the following is not an audit procedure that the independent auditor would perform with respect to
litigation, claims, and assessments?

Inquire of and discuss with management the policies and procedures adopted for litigation, claims, and
assessments.

Obtain from management a description and evaluation of litigation, claims, and assessments that existed at
the balance sheet date.

Obtain assurance from management that if has disclosed all unasserted claims that the lawyer has advised are
probable of assertion and must be disclosed.

Confirm directly with the client’s lawyer that all claims have been recorded in the financial statements.
Ans.
Confirm directly with the client’s lawyer that all claims have been recorded in the financial statements.

The auditor’s judgment concerning the overall fairness of presentation of financial position, results of
operations, and changes in cash flow is applied within the framework of:

quality control.
generally accepted auditing standards which include the concept of materiality.
the auditor’s evaluation of the audited company’s internal control.
generally accepted accounting principles.
Ans.
generally accepted accounting principles.

Which one of the following is not a Field Work Standard?

Adequate planning and supervision.


Due professional care.
Understand the entity and its environment including internal control.
Sufficient appropriate audit evidence.
Ans.
Due professional care.

The objective of the consistency standard is to provide assurance that

There are no variations in the format and presentation of financial statements

Substantially different transactions and events are not accounted for on an identical basis

The auditor is consulted before material changes are made in the application of accounting principles

The comparability of financial statements between periods in not materially affected by changes in accounting
principles without disclosure
Ans.
The comparability of financial statements between periods in not materially affected by changes in accounting
principles without disclosure

Which of the following statements most accurately captures the intent of the standards of field work?

Field work standards are primarily concerned with personal attributes necessary during the conduct of the
audit.

Field work standards provide extensive guidance regarding the conduct of an audit.

Field work standards are primarily directed at the auditor’s planning, understanding of internal control, and
evidence accumulation.

Field work standards are primarily concerned with the conduct of substantive testing as opposed to testing of
internal controls.
Ans.
Field work standards are primarily directed at the auditor’s planning, understanding of internal control, and
evidence accumulation.
Which of the following steps is an auditor most likely to take when using an expert to obtain assurance
concerning a material financial statement assertion?

Qualify his or her audit report with respect to the financial statement area examined by the expert.

Re-perform the work undertaken by the expert to ensure its accuracy

Explain the use of the expert in an explanatory paragraph of the auditor’s report

Assess the competence and objectivity of the expert


Ans.
Assess the competence and objectivity of the expert

In using the work of a specialist, an auditor referred to the specialist’s findings in the auditor’s report. This is
an appropriate reporting practice if the

Auditor is not familiar with the professional certification, personal reputation, or particular competence of the
specialist.

Auditor, as a result of the specialist’s findings, adds an explanatory paragraph emphasizing a matter regarding
the financial statements.

Specialist is aware that his/her work will be used to evaluate the assertions in the financial statements.

Auditor, as a result of the specialist’s findings, decides to indicate a division of responsibility with the
specialist.
Ans.
Auditor, as a result of the specialist’s findings, adds an explanatory paragraph emphasizing a matter regarding
the financial statements.

Which of the following statements is correct about the auditor’s use of the work of an expert?

The expert should not have an understanding of the auditor’s corroborative use of the expert’s findings

The auditor is required to perform substantive procedures to verify the expert’s assumptions and findings

The client should not have an understanding of the nature of the work to be performed by the expert

The auditor should obtain an understanding of the methods and assumptions used by the expert
Ans.
The auditor should obtain an understanding of the methods and assumptions used by the expert

Which of the following least likely requires the services of an expert?

Valuations of certain types of assets like land and buildings.


Legal opinions concerning interpretations of engagements, statutes and regulations.
Determination of amounts using specialized techniques.
Application of accounting methods in computing inventory balances.
Ans.
Application of accounting methods in computing inventory balances.
Which of the following statements is correct concerning an auditor’s use of the work of an expert?

The work of an expert who is related to the client may be acceptable under certain circumstances

If an auditor believes that the determinations made by an expert are unreasonable, only a qualified opinion
may be issued

If there is a material difference between an expert’s findings and the assertions in the financial statements,
only an adverse opinion may be is sued

An auditor may not use a expert in the deter mi nation of physical characteristics relating to inventories.
Ans.
The work of an expert who is related to the client may be acceptable under certain circumstances

Which statement is incorrect regarding the auditor’s use of the work of an expert?

When using the work performed by an expert, the auditor should obtain sufficient appropriate audit evidence
that such work is adequate for the purposes of the audit.

“Expert” means a person or firm possessing special skill, knowledge and experience in a particular field other
than accounting and auditing.

The auditor's education and experience enable the auditor to be knowledgeable about business matters
in general, but the auditor is not expected to have the expertise of a person trained for or qualified to engage
in the practice of another profession or occupation.

When the auditor uses the work of an expert employed by the auditor, that work is used in the employee's
capacity as an assistant on the audit.
Ans.
When the auditor uses the work of an expert employed by the auditor, that work is used in the employee's
capacity as an assistant on the audit.

When the auditor has to determine the need to use the work of an expert, he would least likely consider:

The cost of using the services of an expert.

The quantity and quality of other audit evidence available.

The materiality of the financial statement item being considered.

The risk of misstatement based on the nature and complexity of the matter being considered.
Ans.
The cost of using the services of an expert.

Which of the following statements concerning the auditor’s use of the work of a specialist is true?

If the auditor believes that the determinations made by the specialist are unreasonable, only a qualified
opinion may be expressed.

If the specialist is related to the client, the auditor is still permitted to use the specialist’s findings as
corroborative evidence.
The specialist may not be related to the client.

The specialist is identified in the auditor’s report when the auditor expresses an unqualified opinion.
Ans.
If the specialist is related to the client, the auditor is still permitted to use the specialist’s findings as
corroborative evidence.

An expert may be:

Engaged by the entity


Engaged by the auditor
Employed by the entity
Employed by the auditor
Yes
No
Yes
No
No
Yes
No
Yes
Yes
Yes
No
No
Yes
Yes
Yes
Yes
Ans.
Yes
Yes
Yes
Yes

A person or firm possessing special skill, knowledge and experience in a particular field other than accounting
and auditing is called a/an

Professional
Expert
Consultant
Guru
Ans.
Expert

When obtaining an understanding and performing a preliminary assessment of the internal audit function,
the auditor should consider the internal auditors’
Organizational status Scope of functions Technical competence and due care
Yes Yes Yes

Yes No Yes

Yes No No

No Yes No
Ans.
Yes Yes Yes

An independent appraisal activity established within an entity as a service to the entity is


independent auditing
internal auditing
internal control
external auditing
Ans.
internal auditing

The scope and objectives of internal auditing vary widely and depend on the size and structure of the entity
and the requirements of its management. Ordinarily, internal auditing activities include one or more of the
following, except

Examination of financial and operating information including review of the means to identify, measure,
classify and report such information and specific inquiry into individual items including detailed testing of
transactions and recommending improvements thereto.
Review of the economy, efficiency and effectiveness of operations including non-financial controls of an entity
Establishment of adequate accounting and internal control systems
Review of compliance with laws, regulations and other external requirements and with management policies
and directives and other internal requirements
Ans.
Establishment of adequate accounting and internal control systems

Which of the following is most likely to be an audit area where a CPA places reliance on the work of a client’s
internal auditors?
When assessing control risk
When choosing a random sample for substantive testing
When judgmentally selecting accounts receivable for confirmation
When evaluating a client’s contingent liabilities
Ans.
When assessing control risk

Which of the following is not an appropriate function of internal auditing?


review of accounting and internal control
examination of financial and operating information
review of compliance with laws and regulations
expressing an opinion about the fair presentation of the financial statements.
Ans.
expressing an opinion about the fair presentation of the financial statements.
Which of the following statements in relation to the preliminary assessment of internal auditing is false?

The external auditor should obtain a sufficient understanding of internal audit activities to assist in planning
the audit and developing an effective audit approach
During the course of planning the audit, the external auditor should perform a preliminary assessment of the
internal audit function when it appears that internal auditing is relevant to the external audit of the financial
statements in specific audit areas
Effective internal auditing will often allows a modification in the nature, timing, reduction in the extent and
even eliminate in its entirety some procedures performed by the external auditor
In some cases, after having considered the activities of internal auditing, the external auditor may decide that
internal auditing will have no effect on external audit procedures
Ans.
Effective internal auditing will often allows a modification in the nature, timing, reduction in the extent and
even eliminate in its entirety some procedures performed by the external auditor

Which of the following is considered a primary reason for creating an internal audit department?

To evaluate and improve the effectiveness of control processes.


To ensure the accuracy, reliability, and timeliness of financial and operating data used in management’s
decision making.
To relieve management of the responsibility for establishing effective controls.
To safeguard resources entrusted to the organization.
Ans.
To evaluate and improve the effectiveness of control processes.

Audit sampling involves the

Selection of all items over a certain amount.

Application of audit procedures to less than 100% of items within a class of transactions or an account
balance such that all items have a chance of selection.

Application of audit procedures to all items that comprise a class of transactions or an account balance.

Application of audit procedures to all items over a certain amount and those that are unusual or have a
history of error.
Ans.
Application of audit procedures to less than 100% of items within a class of transactions or an account
balance such that all items have a chance of selection.

The risk that the auditor’s conclusion based on a sample may be different from the conclusion if the entire
population were subjected to the same audit procedure is

Sampling risk
Non-sampling risk
Audit risk
Detection risk
Ans.
Sampling risk

Which of the following is true about sampling and non-sampling risks?


Sampling risk can be reduced by increasing sample size.

Sampling risk cannot be eliminated.

Non-sampling risk can be eliminated by proper engagement planning, supervision, and review.

Non-sampling risk arises from the possibility that the auditor’s conclusion, based on a sample may be
different from the conclusion reached if the entire population were subjected to the same audit procedure.
Ans.
Sampling risk can be reduced by increasing sample size.

Audit sampling is not involved in the following, except

Performing a walkthrough test.


Performing analytical procedures
Selecting the sample without following a structured technique.
Testing controls that leave no audit trail.
Ans.
Selecting the sample without following a structured technique.

Sampling risk that leads the auditor to conclude that controls are more effective than they are actually is

Risk of assessing control risk too low


Risk of assessing control risk too high
Risk of incorrect rejection
Risk of incorrect acceptance
Ans.
Risk of assessing control risk too low

An error that arises from an isolated event that has not recurred other than on specifically identifiable
occasions and is therefore not representative of errors in the population is called
Sampling error.
Non-sampling error.
Anomalous error
Projected error.
Ans.
Anomalous error

Audit sampling is not involved in the following, except

Performing a walkthrough test.


Performing analytical procedures
Selecting the sample without following a structured technique.
Testing controls that leave no audit trail.
Ans.
Selecting the sample without following a structured technique.

The following situations will likely lead the auditor to use 100% testing, except
When the population constitutes a small number of large value items.

When both inherent and control risks are high and other means do not provide sufficient appropriate audit
evidence

When the repetitive nature of a calculation or other process performed by a computer information system
makes a 100% examination cost effective.

When testing controls that leave audit trail.


Ans.
When testing controls that leave audit trail.

The entire set of data from which a sample is selected and about which the auditor wishes to draw
conclusions is

Population
Sample
Sampling unit
Database
Ans.
Population

Which of the following constitutes audit sampling?

Selecting and examining specific items to determine whether or not a particular procedure is being
performed.

Examining items to obtain information about matters such as the client’s business, the nature of transactions,
accounting and internal control systems.

Examining items whose values exceed a certain amount so as to verify a large proportion of the total amount
of an account balance or class of transactions.

Applying audit procedures to less than 100% of items within an account balance or class of transactions such
that all sampling units have a chance of selection.
Ans.
Applying audit procedures to less than 100% of items within an account balance or class of transactions such
that all sampling units have a chance of selection.

Which of the following best illustrates the concept of sampling risk?

A randomly chosen sample may not be representative of the population as a whole on the characteristic of
interest

An auditor may select audit procedures that are not appropriate to achieve the specific objective

An auditor may fail to recognize errors in the documents examined for the chosen sample

The documents related to the chosen sample may not be available for inspection
Ans.
A randomly chosen sample may not be representative of the population as a whole on the characteristic of
interest
The entire set of data about which the auditor wishes to draw conclusions is called

Population.
Sample.
Sampling frame.
Sampling unit.
Ans.
Population.

A monetary amount set by the auditor in respect of which the auditor seeks to obtain an appropriate level of
assurance that the monetary amount set by the auditor is not exceeded by the actual misstatement in the
population is

Tolerable misstatement
Tolerable rate of deviation
Anomaly
Expected misstatement
Ans.
Tolerable misstatement

The decision whether to use statistical or non-statistical sampling depends upon the

Philippine Standards on Auditing


Auditor’s judgment
Size of the population
Generally Accepted Auditing Standards
Ans.
Auditor’s judgment

An advantage of using statistical over non-statistical sampling methods in tests of controls is that the
statistical methods

Can more easily convert the sample into a dual-purpose test useful for substantive testing
Eliminate the need to use judgment in determining appropriate sample sizes
Afford greater assurance than a non-statistical sample of equal size
Provide an objective basis for quantitatively evaluating sample risk
Ans.
Provide an objective basis for quantitatively evaluating sample risk

The likelihood of assessing control risk too low is the risk that the sample selected to test controls

Does not support the tolerable misstatement for some or all of management’s assertions.

Does support the auditor’s planned assessed level of control risk when the true operating effectiveness of the
control does not justify such an assessment.

Contains misstatements that could be material to the financial statements when aggregated with
misstatements in other account balances.
Contains proportionately more deviations from prescribed internal control policies or procedures than
exist in the population.
Ans.
Does support the auditor’s planned assessed level of control risk when the true operating effectiveness of the
control does not justify such an assessment.

The size of a sample designed for dual-purpose testing should be

The larger of the samples that would otherwise have been designed for the two separate purposes

The smaller of the samples that would otherwise have been designed for the two separate purposes

The combined total of the samples that would otherwise have been designed for the two separate purposes

More than the larger of the samples that would otherwise have been designated for the two separate
purposes, but less than the combined total of the samples that would otherwise have been designed for the
two separate purposes
Ans.
The larger of the samples that would otherwise have been designed for the two separate purposes

Population, as defined in PSA 530, means the entire set of data from which a sample is selected and about
which the auditor wishes to draw conclusions. It is important for the auditor to ensure that the population is

I. Appropriate to the objective of the audit procedure.


II. Complete.

I only
Both I and II
II only
Neither I nor II
Ans.
Both I and II

The diagram below depicts the auditor’s estimated maximum deviation rate compared with the tolerable rate
and also depicts the true population deviation rate compared with the tolerable rate.

True State of Population


Auditor’s
Estimate Deviation Rate Deviation Rate
Based on Is less than Exceeds
Sample Results Tolerable Rate Tolerable Rate

Maximum
Deviation Rate I III
Is Less than
Tolerable Rate

Maximum
Deviation Rate II IV
Exceeds
Tolerable Rate

As a result of tests of controls, the auditor assesses control risk higher than necessary and thereby increases
substantive testing. This is illustrated by

I
II
III
IV
Ans.
II

While performing a substantive test of details during an audit, the auditor determined that the sample results
supported the conclusion that the recorded account balance was materially misstated. It was, in fact, not
materially misstated. This situation illustrates the risk of

Assessing control risk too low.


Assessing control risk too high.
Incorrect rejection.
Incorrect acceptance.
Ans.
Incorrect rejection.

At times a sample may indicate that the auditor’s assessed level of control risk for a given control is
reasonable when, in fact, the true compliance rate does not justify the assessed level.
This situation illustrates the risk of

Assessing control risk too low


Assessing control risk too high
Incorrect precision
Incorrect rejection
Ans.
Assessing control risk too low

Conducting a substantive test of an account balance, an auditor hypothesis that no material misstatement
exists. The risk that sample results will support the hypothesis when a material misstatement actually does
exist is the risk of

Incorrect rejection.
Alpha error.
Incorrect acceptance.
Type I error.
Ans.
Incorrect acceptance.

The tolerable rate of deviations for a test of controls is generally

Lower than the expected rate of errors in the related accounting population.
Higher than the expected rate of errors in the related accounting records.
Identical to the expected rate of errors in the related accounting records.
Unrelated to the expected rate of errors in the related accounting records.
Ans.
Higher than the expected rate of errors in the related accounting records.

Which of the following combinations results in a decrease in sample size in a sample for attributes?
1) Risk of assessing control risk too low
2) Tolerable rate
3) Expected population deviation rate

1) Increase 2) Decrease 3) Increase


1) Decrease 2) Increase 3) Decrease
1) Increase 2) Increase 3) Decrease
1) Increase 2) Increase 3) Increase
Ans.
1) Increase 2) Increase 3) Decrease\

Which of the following sample planning factors would influence the sample size for a substantive test of
details for a specific account?
Expected amount of misstatements Measure of tolerable misstatement
No No
Yes Yes
No Yes
Yes No
Ans.
Yes Yes

A number of factors influence the sample size for a substantive test of details of an account balance. All other
factors being equal, which of the following would lead to a larger sample size?

Greater reliance on internal controls


Greater reliance on analytical procedures
Smaller measure of tolerable misstatement
Smaller reliance on analytical procedures
Ans.
Smaller measure of tolerable misstatement

Which of the following combinations results in a decrease in sample size in a sample for attributes?
Risk of assessing control risk too low Tolerable rate Expected population deviation rate
Increase Decrease Increase
Decrease Increase Decrease
Increase Increase Decrease
Increase Increase Increase
Ans.
Increase Increase Decrease

Which of the following courses of action would an auditor most likely follow in planning a sample of cash
disbursements if the auditor is aware of several unusually large cash disbursements?
Set the tolerable rate deviation at a lower level than originally planned.

Stratify the cash disbursements population so that the unusually large disbursements are selected.

Increase the sample size to reduce the effect of the unusually large disbursements.

Continue to draw new samples until all the unusually large disbursements appear in the sample.
Ans.
Stratify the cash disbursements population so that the unusually large disbursements are selected.

The entire set of data about which the auditor wishes to draw conclusions is called

Population.
Sample.
Sampling frame.
Sampling unit.
Ans.
Population.

How would increases in tolerable misstatement and assessed level of control risk affect the sample size in a
substantive test of details?
Increase in tolerable misstatement Increase in assessed level of control risk
Increase sample size Increase sample size
Increase sample size Decrease sample size
Decrease sample size Increase sample size
Decrease sample size Decrease sample size
Ans.
Decrease sample size Increase sample size

Which of the following courses of action would an auditor most likely follow in planning a sample of cash
disbursements if the auditor is aware of several unusually large cash disbursements?

Set the tolerable rate deviation at a lower level than originally planned.

Stratify the cash disbursements population so that the unusually large disbursements are selected.

Increase the sample size to reduce the effect of the unusually large disbursements.

Continue to draw new samples until all the unusually large disbursements appear in the sample.
Ans.
Stratify the cash disbursements population so that the unusually large disbursements are selected.

Using statistical sampling to assist in verifying the year-end accounts payable balance, an auditor has
accumulated the following data:
Number of Book Balance Determined by
Accounts Balance the Auditor
Population 4,100 P5,000,000 ?
Sample 200 P 250,000 P300,000

Using the ratio estimation technique, the auditor’s estimate of year-end accounts payable balance is
P6,150,00
P6,000,000
P5,125,000
P5,050,000
Ans.
P6,000,000

An auditor is applying a difference estimation sampling plan. Recorded book value is P1,000,000 and the
auditor estimates a P75,000 understatement difference. In this case, the auditor’s estimated population value
is

P925,000
P1,075,000
P1,000,000
P1,025,000
Ans.
P1,075,000

Which of the following statistical selection techniques is least desirable for use by an auditor?

Systematic selection
Stratified selection
Block selection
Sequential selection
Ans.
Stratified selection

When the auditor goes through a population and selects items for the sample without regard to their size,
source, or other distinguishing characteristics, it is called

Block selection
Random selection
Systematic selection
Haphazard selection
Ans.
Haphazard selection

A sample in which every possible combination of items in the population has an equal chance of constituting
the sample is a

Representative sample
Statistical sample
Random sample
Judgment sample
Ans.
Random sample

If the auditor is concerned that a population may contain exceptions, the determination of a sample size
sufficient to include at least one such exception is a characteristic of
Discovery sampling
Variables sampling
Random sampling
Peso-unit sampling
Ans.
Discovery sampling

An auditor wishes to sample 200 sales receipts from a population of 5,000 receipts issued during the last year.
The receipts have preprinted serial numbers and are arranged in chronological (and thus serial number)
order. The auditor randomly chooses a receipt from the first 25 receipts and then selects every 25th receipt
thereafter. The sampling procedure described here is called

Systematic random sampling.


Monetary-unit sampling.
Judgment interval sampling
Variables sampling.
Ans.
Systematic random sampling.

Which of the following statistical sampling plans does not use a fixed sample size for tests of controls?

Peso-unit sampling
Sequential sampling
PPS sampling
Variables sampling
Ans.
Sequential sampling

Which of the following sampling methods would be used to estimate a numerical measurement of a
population, such as a peso value?

Attribute sampling
Stop-or-go sampling
1. Variables sampling
Random-number sampling
Ans.
1. Variables sampling

In statistical sampling methods used in substantive testing, when would an auditor most likely stratify a
population into meaningful groups?

If the population has highly variable recorded amounts.


If probability proportional to size sampling is used.
If the auditor’s estimated tolerable misstatement is extremely small.
If the standard deviation of recorded amounts is relatively small.
Ans.
If the population has highly variable recorded amounts.

An auditor is applying mean-per-unit estimation. Assuming estimated audited value is P950,000, the achieved
allowance for sampling risk is P75,000, and recorded book value is P925,000, what is the auditor's
conclusion?
Recorded book value is not likely misstated by a material amount.

Recorded book value is misstated by a material amount.

Recorded book value is not likely misstated by a material amount, assuming the
client records an adjusting journal entry equal to the allowance for sampling risk.

There is insufficient evidence to reach a conclusion.


Ans.
Recorded book value is not likely misstated by a material amount.

An auditor is evaluating the results of a variables sampling plan. Which of the following is not relevant to the
auditor's judgment about the sample?

Management's explanations for why errors in the sample occurred.


Projecting the sample error to the population.
Considering the effects of sampling risk.
Qualitative information that lends insight into errors found.
Ans.
Management's explanations for why errors in the sample occurred.

Tests of controls provide reasonable assurance that controls are applied as prescribed. A sampling method
that is useful when testing controls is:

Nonstatistical sampling
Attribute estimation sampling
Discovery sampling
Stratified random sampling
Ans.
Attribute estimation sampling

For which of the following audit tests would an auditor most likely use attribute sampling?

Making an independent estimate of the amount of a FIFO inventory.


Examining invoices in support of the valuation of fixed asset additions.
Selecting accounts receivable for confirmation of account balances.
Inspecting employee time cards for proper approval by supervisors.
Ans.
Inspecting employee time cards for proper approval by supervisors.

Assessing control risk too high is the risk that the sample

Does not support tolerable error for some or all of management’s assertions.

Contains proportionately more deviations from prescribed control procedures than actually exist in the
population as a whole.

Contains monetary misstatements that could be material to the financial statements when aggregated with
misstatements in other account balances or classes of transactions.
Contains proportionately fewer deviations from prescribed control procedures than actually exist in the
population as a whole.
Ans.
Contains proportionately more deviations from prescribed control procedures than actually exist in the
population as a whole.

When sampling for attributes, which of the following would decrease sample size?

Risk of assessing Tolerable rate Expected population


Control risk too low of deviation deviation rate
Increase Decrease Increase
Decrease Increase Decrease
Increase Increase Decrease
Increase Increase Increase
Ans.
Increase Increase Decrease

The estimate of the population peso value using ratio estimation is __________.

P4,727,273
P5,500,000
P5,700,000
P5,720,000
Ans.
P5,720,000

An advantage of statistical sampling over nonstatistical sampling is that statistical sampling helps an auditor
to

Minimize the failure to detect errors and frauds.


Eliminate nonsampling risk.
Reduce the level of audit risk and materiality to a relatively low amount.
Measure the sufficiency of the evidential matter obtained.
Ans.
Measure the sufficiency of the evidential matter obtained.

The tolerable rate of deviation for tests of controls necessary to justify a control risk assessment depends
primarily on which of the following?

The cause of errors.


The extent of reliance to be placed on the procedures.
The amount of any substantive errors.
The limit used in audits of similar clients.
Ans.
The extent of reliance to be placed on the procedures.

The diagram below depicts the auditor’s estimated maximum deviation rate compared with the tolerable rate,
and also depicts the true population deviation rate compared with the tolerable rate.
True State of Population

Auditor’s Estimate Based on


Sample Results Deviation Rate Exceeds
Tolerable Rate Deviation Rate is Less
Than Tolerable Rate
Maximum Deviation Rate Exceeds Tolerable Rate
I
III
Maximum Deviation Rate is Less Than Tolerable Rate
II
IV

I.
II.
III.
IV.
Ans.
III.

One of the audit objectives for a manufacturing company is to verify that all rework is reviewed by the
production engineer. Which of the following audit procedures would provide the best evidence for meeting
this objective?

Trace a sample of entries in the rework log to remedial action taken.


Trace a sample of rework orders to entries in the rework log.
Trace a sample of entries in the review log to rework orders.
Trace a sample of rework orders to entries in the review log.
Ans.
Trace a sample of rework orders to entries in the review log.

When using classical variables sampling for estimation, an auditor normally evaluates the sampling results by
calculating the possible error in either direction. This statistical concept is known as

Precision.
Reliability.
Projected error.
Standard deviation.
Ans.
Precision.

An auditor is preparing to sample accounts receivable for overstatement. A statistical sampling method that
automatically provides stratification when using systematic selection is

Attribute sampling.
Ratio-estimation sampling.
Probability proportionate to size sampling
Mean-per-unit (MPU) sampling.
Ans.
Probability proportionate to size sampling
Which of the following most likely would be an advantage in using classical variables sampling rather than
probability-proportional-to-size (PPS) sampling?

An estimate of the standard deviation of the population’s recorded amounts is not required.

The auditor rarely needs the assistance of a computer program to design an efficient sample.

Inclusion of zero and negative balances generally does not require special design considerations.

Any amount that is individually significant is automatically identified and selected.


Ans.
Inclusion of zero and negative balances generally does not require special design considerations.

Which of the following sampling methods is used to estimate a numerical measurement of a population, such
as a peso value?

Attribute sampling.
Stop-or-go sampling.
Variables sampling.
Random-number sampling.
Ans.
Variables sampling.

If a selected random number matches the number of a voided voucher, the voucher ordinarily should be
replaced by another voucher in the sample if the voucher

Constitutes a deviation
Has been properly voided
Cannot be located
Represents an immaterial peso amount
Ans.
Has been properly voided

An auditor uses a number of techniques to select samples. A frequently, and appropriately, used technique is
random selection. In which of the following situations would random selection be least justified? The auditor
needs to

Test sales transactions to determine that they were properly authorized and are supported by shipping
documents.

Confirm accounts receivable and has already selected the 10 largest accounts for confirmation. The remaining
accounts are not numbered. The auditor only has a computer listing of the accounts in alphabetical order
approximately 250 pages long with 50 account balances on every page.

Obtain evidence on the proper sales cut-off by sampling items from the monthly sales journal to determine if
the items were recorded in the correct time period.

Test the perpetual inventory records to ensure that the sample covers the largest dollar value items in the
account.
Ans.
Obtain evidence on the proper sales cut-off by sampling items from the monthly sales journal to determine if
the items were recorded in the correct time period.

Which of the following engagement objectives will be accomplished by tracing a sample of accounts receivable
debit entries to customer invoices and related shipping documents?

Sales are properly recorded.


Sales are billed at the correct prices.
Accounts receivable represent valid sales.
Customer credit is approved.
Ans.
Accounts receivable represent valid sales.

An auditor is testing internal control procedures that are evidenced on an entity’s vouchers by matching
random numbers with voucher numbers. If a random number matches the number of a voided voucher, that
voucher ordinarily should be replaced by another voucher in the random sample if the voucher

Constitutes a deviation
Has been properly voided
Cannot be located
Represents an immaterial amount
Ans.
Has been properly voided

The likelihood of assessing control risk too high is the risk that the sample selected to test controls

Does not support the auditor’s planned assessed level of control risk when the true operating effectiveness of
the control justifies such an assessment.

Contains misstatements that could be material to the financial statements when aggregated with
misstatements in other account balances or transaction classes.

Contains proportionately fewer monetary errors or deviations from prescribed internal control policies or
procedures than exist in the balance or class as a whole.

Does not support the tolerable misstatement for some or all of management’s assertions
Ans.
Does not support the auditor’s planned assessed level of control risk when the true operating effectiveness of
the control justifies such an assessment.

When using classical variables sampling for estimation, an auditor normally evaluates the sampling results by
calculating the possible error in either direction. This statistical concept is known as

Precision.
Reliability.
Projected error.
Standard deviation.
Ans.
Precision.
When would difference estimation or ratio estimation sampling methods be inappropriate?

If differences between the book values and audit values of a population are rare.

If the average difference between the audit value and book value of a population is small.

If differences between the book value and audit value of a population are numerous.

If the average difference between the audit value and book value of a population is large.
Ans.
If differences between the book values and audit values of a population are rare.

The estimate of the population peso value using difference estimation sampling is __________.

P4,700,000
P5,500,000
P5,680,000
P5,700,000
Ans.
P5,700,000

If the auditor is concerned that a population may contain exceptions, the determination of a sample size
sufficient to include at least one such exception is a characteristic of

Discovery sampling
Variable sampling
random sampling
PPS
Ans.
Discovery sampling

The risk of incorrect acceptance and the risk of assessing control risk too low relate to the

Preliminary estimates of materiality levels


Allowable risk of tolerable error
Efficiency of the audit
Effectiveness of the audit
Ans.
Effectiveness of the audit

Since auditors are interested in the occurrence of exceptions in population, they refer to the occurrence as

Exception rate
Population rate
Deviation rate
Confidence level
Ans.
Deviation rate
As a result of sampling procedures applied as tests of controls, an auditor incorrectly assesses control risk
lower than appropriate. The most likely explanation for this situation is that

The deviation rates of both the auditor’s sample and the population exceed the tolerable rate.

The deviation rates of both the auditor’s sample and the population are less than the tolerable rate.

The deviation rate in the auditor’s sample is less than the tolerable rate, but the deviation rate in the
population exceeds the tolerable rate.

The deviation rate in the auditor’s sample exceeds the tolerable rate, but the deviation rate in the population
is less than the tolerable rate.
Ans.
The deviation rate in the auditor’s sample is less than the tolerable rate, but the deviation rate in the
population exceeds the tolerable rate.

“Whenever a sample is taken, there is a risk that the quantitative conclusions about the population will be
incorrect.”

This is always true.


This is always true unless 100 percent of the population is tested.
This is true for statistical sampling, but not for non-statistical sampling.
This is true for non-statistical sampling but not for statistical sampling.
Ans.
This is always true unless 100 percent of the population is tested.

The risk of incorrect acceptance and the likelihood of assessing control risk too low relate to the

Allowable risk of tolerable misstatement


Preliminary estimates of materiality levels
Efficiency of the audit
Effectiveness of the audit
Ans.
Effectiveness of the audit

An advantage of statistical over nonstatistical sampling methods in tests of controls is that the statistical
methods

Afford greater assurance than a nonstatistical sample of equal size.

Provide an objective basis for quantitatively evaluating sampling risks.

Can more easily convert the sample into a dual-purpose test useful for substantive testing.

Eliminate the need to use judgment in determining appropriate sample sizes.


Ans.
Provide an objective basis for quantitatively evaluating sampling risks.

Per PSA 530, projection of sample results to the population, is only necessary in the case of
A. Deviations (test of controls)
b. Misstatement (test of details)

A only
B only
A or B
Neither A or B
Ans.
Neither A or B

When an auditor’s sampling objective is to obtain a measurable assurance that a sample will contain at least
one occurrence of a specific critical exception existing in a population, the sampling approach to use is

Random.
Discovery.
Probability proportional to size.
Variables.
Ans.
Discovery.

The next 3 questions are based on the following information:


An auditor has obtained the following data by selecting a random sample from an inventory population:

Number Audited Carrying


Of Items Value Amount
Sample 200 P220,000 P 200,000
Population 5,000 ? P5,200,000

The estimate of the population peso value using mean-per-unit sampling is __________.

P5,000,000
P5,420,000
P5,500,000
P5,720,000
Ans.
P5,500,000

As a result of sampling procedures applied as tests of controls, and auditor incorrectly assess control risk
lower than appropriate. The most likely explanation for this situation is that

The deviation rates of both the auditor's sample and the population exceed the tolerable rate

The deviation rate of both the auditor's sample and the population is less than the tolerable rate

The deviation rate in the auditor's sample is less than the tolerable rate, but the deviation rate in the
population exceeds the tolerable rate

The deviation rate in the auditor's sample exceeds the tolerable rate, but the deviation rate in the population
is less than the tolerable rate
Ans.
The deviation rate in the auditor's sample is less than the tolerable rate, but the deviation rate in the
population exceeds the tolerable rate

As lower acceptable levels of both audit risk and materiality are established, the auditor should plan more
work on individual accounts to

Find smaller misstatements.


Find larger misstatements.
Increase the tolerable misstatement in the accounts.
Decrease the risk of assessing control risk too low.
Ans.
Find smaller misstatements.

After partially completing a control review of the accounts payable department, an auditor suspects that some
type of fraud has occurred. To ascertain whether the fraud is present, the best sampling approach is to use

Simple random sampling to select a sample of vouchers processed by the department during the past year.

Probability-proportional-to-size sampling to select a sample of vouchers processed by the department during


the past year.

Discovery sampling to select a sample of vouchers processed by the department during the past year.

Judgmental sampling to select a sample of vouchers processed by clerks identified by the department
manager as acting suspiciously.
Ans.
Discovery sampling to select a sample of vouchers processed by the department during the past year.

In determining the sample size for a test of controls, an auditor should consider the likely rate of deviations,
the allowable risk of assessing control risk too low, and the

Tolerable deviation rate


Risk of incorrect acceptance
Nature and cause of deviations
Population size
Ans.
Tolerable deviation rate

Statistical sampling may be applied to test controls when a client’s control procedures

Depend primarily on segregation of duties.


Are carefully reduced to writing and are included in client accounting manuals.
Leave an audit trail as evidence of compliance.
Enable the detection of fraud.
Ans.
Leave an audit trail as evidence of compliance.

As a result of sampling procedures applied as tests of controls, and auditor incorrectly assess control risk
lower than appropriate. The most likely explanation for this situation is that
The deviation rates of both the auditor's sample and the population exceed the tolerable rate

The deviation rate of both the auditor's sample and the population is less than the tolerable rate

The deviation rate in the auditor's sample is less than the tolerable rate, but the deviation rate in the
population exceeds the tolerable rate

The deviation rate in the auditor's sample exceeds the tolerable rate, but the deviation rate in the population
is less than the tolerable rate
Ans.
The deviation rate in the auditor's sample is less than the tolerable rate, but the deviation rate in the
population exceeds the tolerable rate

An auditor desired to test credit approval on 10,000 sales invoices processed during the year. The auditor
designed a statistical sample that would provide 1% risk of assessing control risk too low (99% confidence)
that not more than 7% of the sales invoices lacked approval. The auditor estimated from previous experience
that about 2 1/2% of the sales invoices lacked approval. A sample of 200 invoices was examined and 7 of them
were lacking approval. The auditor then determined the achieved upper precision limit to be 8%. In the
evaluation of this sample, the auditor decided to increase the level of the preliminary assessment of control
risk because the

Tolerable rate (7%) was less than the achieved upper precision limit (8%)

Expected deviation rate (7%) was more than the percentage of errors in the sample (3 1/2%)

Achieved upper precision limit (8%) was more than the percentage of errors in the sample (3 1/2%)

Expected deviation rate (2 1/2%) was less than the tolerable rate (7%)
Ans.
Tolerable rate (7%) was less than the achieved upper precision limit (8%)

In applying variables sampling, an auditor attempts to

Estimate a qualitative characteristic of interest.


Determine various rates of occurrence for specified attributes.
Discover at least one instance of a critical deviation.
Predict a monetary population value within a range of precision.
Ans.
Predict a monetary population value within a range of precision.

A statistical sampling technique that will minimize sample size whenever a low deviation rate is expected is

Ratio-estimation sampling.
Difference-estimation sampling.
Stratified mean-per-unit sampling
Stop-or-go sampling.
Ans.
Stop-or-go sampling.
Each time an auditor draws a conclusion based on evidence from a sample, an additional risk, sampling risk, is
introduced. An example of sampling risk is

Projecting the results of sampling beyond the population tested.


Properly applying an improper audit procedure to sample data.
Improperly applying a proper audit procedure to sample data.
Drawing an erroneous conclusion from sample data.
Ans.
Drawing an erroneous conclusion from sample data.

In estimation sampling for variables, which of the following must be known in order to estimate the
appropriate sample size required to meet the auditor’s needs in a given situation?

The qualitative aspects of errors


The total peso amount of the population
The acceptable level of risk
The estimated rate of misstatements in the population
Ans.
The total peso amount of the population

Conducting a substantive test of an account balance, an auditor hypothesis that no material misstatement
exists. The risk that sample results will support the hypothesis when a material misstatement actually does
exist is the risk of

Incorrect rejection.
Alpha error.
Incorrect acceptance.
Type I error.
Ans.
Incorrect acceptance.

After issuing a report, an auditor has no obligation to make continuing inquiries or perform other procedures
concerning the audited financial statements, unless

Final determinations or resolutions are made of contingencies that had been disclosed in the financial
statements.

Information about an event that occurred after the date of the auditor’s report comes to the auditor’s
attention.

The control environment changes after issuance of the report.

Information, which existed at the report date and may affect the report, comes to the auditor’s attention.
Ans.
Information, which existed at the report date and may affect the report, comes to the auditor’s attention.

Which of the following statements best describes the “date of the financial statements?”

The date on which those with the recognized authority assert that they have prepared the entity’s complete
set of financial statements, including the related notes, and that they have taken responsibility for them.
The date that the auditor’s report and audited financial statements are made available to third parties.

The date of the end of the latest period covered by the financial statements, which is normally the date of the
most recent balance sheet in the financial statements subject to audit.

The date on which the auditor has obtained sufficient appropriate audit evidence on which to base the
opinion on the financial statements.
Ans.
The date of the end of the latest period covered by the financial statements, which is normally the date of the
most recent balance sheet in the financial statements subject to audit.

After an audit report containing an unmodified opinion on a client’s financial statements was issued, the client
decided to sell the shares of a subsidiary that accounts for 30% of its revenue and 25% of its net income. The
auditor should

Determine whether the information is reliable and, if determined to be reliable, request that re vised financial
statements be issued

Notify the entity that the auditor’s report may no longer be associated with the financial statements

Describe the effects of this subsequently discovered in formation in a communication with persons known to
be relying on the financial statements

Take no action because the auditor has no obligation to make any further inquiries
Ans.
Take no action because the auditor has no obligation to make any further inquiries

Which of the following procedures would an auditor most likely perform to obtain evidence about an entity’s
subsequent events?

Reconcile bank activity for the month after the reporting date with cash activity reflected in the accounting
records.

Examine on a test basis the purchase invoices and receiving reports for several days after the inventory date.

Review the treasure’s monthly reports on temporary investment owned, purchased, and sold.

Obtain a letter from the entity’s attorney describing any pending litigation, unasserted claims, or loss
contingencies.
Ans.
Obtain a letter from the entity’s attorney describing any pending litigation, unasserted claims, or loss
contingencies.

PSA 570 (Going Concern) states that a fundamental principle in the preparation of financial statements is the
going concern assumption. Under this assumption, an entity is ordinarily viewed as continuing in business
for the foreseeable future with neither the intention nor the necessity of liquidation, ceasing trading or
seeking protection from creditors pursuant to laws and regulations. The responsibility to make an
assessment of an entity’s ability to continue as a going concern rests with the

Auditor
Entity’s management
SEC
Entity’s creditors
Ans.
Entity’s management

Which of the following statements best describes the auditor’s responsibility concerning the appropriateness
of the going concern assumption in the preparation of the financial statements?

The auditor’s responsibility is to make a specific assessment of the entity’s ability to continue as a going
concern.

The auditor’s responsibility is to predict future events or conditions that may cause the entity to cease to
continue as a going concern.

The auditor’s responsibility is to consider the appropriateness of management’s use of the going concern
assumption and consider whether there are material uncertainties about the entity’s ability to continue as a
going concern that need to be disclosed in the financial statements.

The auditor’s responsibility is to give a guarantee in the audit report that the entity has the ability to continue
as a going concern.
Ans.
The auditor’s responsibility is to consider the appropriateness of management’s use of the going concern
assumption and consider whether there are material uncertainties about the entity’s ability to continue as a
going concern that need to be disclosed in the financial statements.

When an auditor concludes that there is substantial doubt about a continuing audit client’s ability to continue
as a going concern for a reasonable period of time, the auditor’s responsibility is to

Consider the adequacy of disclosure about the client’s possible inability to continue as a going concern.

Issue a qualified or adverse opinion, depending upon materiality, due to the possible effects on the financial
statements.

Report to the client’s audit committee that management’s accounting estimates may need to be adjusted.

Reissue the prior year’s auditor’s report and add an emphasis of matter paragraph that specifically refers to
“substantial doubt” and “going concern.”
Ans.
Consider the adequacy of disclosure about the client’s possible inability to continue as a going concern.

By definition, subsequent events for reporting purposes occur between:

the balance sheet date and the report date.


the report date and the date the report is issued.
the date the report is approved and the date the report is issued.
the balance sheet date and the date the report is issued.
Ans.
the balance sheet date and the report date.

Which of the following audit procedures would most likely assist an auditor in identifying conditions and
events that may indicate there could be substantial doubt about an entity’s ability to continue as a going
concern?
Confirmation of bank balances.
Confirmation of accounts receivable from major customers.
Reconciliation of interest expense with debt outstanding.
Review of compliance with terms of debt agreements.
Ans.
Review of compliance with terms of debt agreements.

As used in PSA 560 (Subsequent Events), the term “subsequent events” refers to

I. Events occurring between the date of the financial statements and the date of the auditor’s report.
II. Facts discovered after the date of the auditor’s report.

I only.
II only.
Both I and II.
Neither I nor II.
Ans.
Both I and II.

Which of the following procedures would an auditor most likely perform to obtain evidence about the
occurrence of subsequent events?

Inquiring as to whether any unusual adjustments were made after the date of the financial statements.

Confirming a sample of material accounts receivable established after the date of the financial statements.

Comparing the financial statements being reported on with those of the prior period.

Investigating personnel changes in the accounting department occurring after the date of the financial
statements.
Ans.
Inquiring as to whether any unusual adjustments were made after the date of the financial statements.

Which of the following conditions or events most likely would cause an auditor to have substantial doubt
about an entity’s ability to continue as a going concern?

Cash flows from operating activities are negative.


Stock dividends replace annual cash dividends.
Significant related party transactions are pervasive.
Research and development projects are postponed.
Ans.
Cash flows from operating activities are negative.

Harold, CPA, believes there is substantial doubt about the ability of Jersamtan Co. to continue as a going
concern for a reasonable period of time. In evaluating Jersamtan’s plans for dealing with the adverse effects of
future conditions and events, Harold most likely would consider, as a mitigating factor, Jersamtan’s plans to

Postpone expenditures for research and development projects.


Purchase production facilities currently being leased from a related party.
Strengthen internal controls over cash disbursements.
Discuss with lenders the terms of all debt and loan agreements.
Ans.
Postpone expenditures for research and development projects.

Which of the following events occurring after the issuance of an auditor’s report most likely would cause the
auditor to make further inquiries about the previously issued financial statements?

A technological development that could affect the entity’s future ability to continue as a going concern.

The entity’s sale of a subsidiary that accounts for 30% of the entity’s consolidated sales.

The discovery of information regarding a contingency that existed before the financial statements were
issued.

The final resolution of a lawsuit disclosed in the notes to the financial statements.
Ans.
The discovery of information regarding a contingency that existed before the financial statements were
issued.

Which of the following statements best expresses the auditor’s responsibility with respect to facts discovered
after the date of the auditor’s report but before the date the financial statements are issued?

The auditor should amend the financial statements.

If the facts discovered will materially affect the financial statements, the auditor should issue a new report
which contains either a qualified opinion or an adverse opinion.

The auditor should consider whether the financial statements need amendment, discuss the matter with
management, and consider taking actions appropriate in the circumstances.

The auditor should withdraw from the engagement.


Ans.
The auditor should consider whether the financial statements need amendment, discuss the matter with
management, and consider taking actions appropriate in the circumstances.

After determining that a related party transaction has, in fact, occurred, an auditor
should Obtain an understanding of the business purpose of the transaction.

Substantiate that the transaction was consummated on terms equivalent to an arm’s-length transaction.

Add a separate paragraph to the auditor’s report to explain the transaction.

Perform analytical procedures to verify whether similar transactions occurred, but were not recorded.
Ans.
Obtain an understanding of the business purpose of the transaction.

The auditor should review information provided by those charged with governance and management
identifying

I. The names of all known related parties.


II. Related party transactions.

I only.
II only.
Both I and II.
Neither I nor II.
Ans.
Both I and II.

When auditing related party transactions, the auditor place primary emphasis on

determining the accuracy and classification of the related party transactions


testing the existence of the related parties
proper accounting for, and disclosure of, the related party transactions
eliminating the effects of related party transactions
Ans.
proper accounting for, and disclosure of, the related party transactions

An auditor searching for related party transactions should obtain an understanding of each subsidiary’s
relationship to the total entity because

This may permit the audit of intercompany account balances to be performed as of concurrent dates.

This may reveal whether particular transactions would have taken place if the parties had not been related.

The business structure may be deliberately designed to obscure related party transactions.

Intercompany transactions may have been consummated on terms equivalent to arm’s-length transactions.
Ans.
The business structure may be deliberately designed to obscure related party transactions.

Which of the following events most likely indicates the existence of related parties?

Making a loan without scheduled terms for repayment of the funds.


Discussing merger terms with a company that is a major competitor.
Selling real estate at a price that differs significantly from its book value.
Borrowing a large sum of money at a variable rate of interest.
Ans.
Making a loan without scheduled terms for repayment of the funds.

Which of the following auditing procedures most likely would assist an auditor in identifying related-party
transactions?

Inspecting correspondence with lawyers for evidence of unreported contingent liabilities.

Vouching accounting records for recurring transactions recorded just after the balance sheet date.

Reviewing confirmations of loans receivable and payable for indications of guarantees.

Performing analytical procedures for indications of possible financial difficulties.


Ans.
Reviewing confirmations of loans receivable and payable for indications of guarantees.

After determining that a related party transaction has, in fact, occurred, an auditor should

Obtain an understanding of the business purpose of the transaction.

Substantiate that the transaction was consummated on terms equivalent to an arm’s-length transaction.

Add a separate paragraph to the auditor’s report to explain the transaction.

Perform analytical procedures to verify whether similar transactions occurred, but were not recorded.
Ans.
Obtain an understanding of the business purpose of the transaction.

Which of the following statements extracted from a client’s lawyer’s letter concerning litigation, claims, and
assessments most likely would cause the auditor to request clarification?

“I believe that the action can be settled for less than the damages claimed.”
“I believe that the company will be able to defend this action successfully.”
“I believe that the plaintiff’s case against the company is without merit.”
“I believe that the possible liability to the company is nominal in amount.”
Ans.
“I believe that the action can be settled for less than the damages claimed.”

The letter of audit inquiry should be

Prepared and sent by the auditor.


Prepared by management and sent by the auditor.
Prepared and sent by management.
Prepared by the auditor and sent by management.
Ans.
Prepared by management and sent by the auditor.

Management’s refusal to give the auditor permission to communicate with the entity’s legal counsel is most
likely to lead to

An adverse opinion.
A qualified opinion or an adverse opinion.
An unqualified opinion.
A qualified opinion or a disclaimer of opinion.
Ans.
A qualified opinion or a disclaimer of opinion.

The primary reason an auditor requests that letters of inquiry be sent to a client’s attorneys is to provide the
auditor with

A description and evaluation of litigation, claims, and assessments that existed at the balance sheet date.

The attorneys’ opinions of the client’s historical experiences in recent similar litigation.
Corroboration of the information furnished by management about litigation, claims, and assessments.

The probable outcome of asserted claims and pending or threatened litigation.


Ans.
Corroboration of the information furnished by management about litigation, claims, and assessments.

In which of the following circumstances would an auditor most likely meet with the client’s legal counsel to
discuss the likely outcome of the litigation and claims?

I. The auditor determines that the matter is a significant risk.


II. There is a disagreement between management and the entity’s legal counsel.
III. The subject matter of the litigation is complex.

I and II only.
II and III only.
I and III only.
I, II, and III.
Ans.
I, II, and III.

The primary source of information to be reported about litigation, claims, and assessments is the

Independent auditor
Client’s management
Court records
Client’s lawyer
Ans.
Client’s management

The auditor is required to obtain audit evidence that management

I. Acknowledges its responsibility for the fair presentation of the financial statements in accordance with
applicable financial reporting framework.
II. Has approved the financial statements.

I only.
II only.
Both I and II.
Neither I nor II.
Ans.
Both I and II.

The auditor should consider the status of legal matters up to the

Balance sheet date.


Date of the auditor’s report.
Date of approval of the financial statements.
Date of issuance of the financial statements.
Ans.
Date of the auditor’s report.
The refusal of a client’s lawyer to provide a representation on the legality of a particular act committed by the
client is ordinarily

Proper grounds to withdraw from the engagement.

Insufficient reason to modify the auditor’s report because of the lawyer’s obligation of confidentiality.

Considered to be a scope limitation.

Sufficient reason to issue a “subject to” opinion.


Ans.
Considered to be a scope limitation.

When considering the use of management’s written representations as audit evidence about the
completeness assertion, an auditor should understand that such representations

Constitute sufficient appropriate audit evidence to support the assertion when considered in combination
with a sufficiently low assessed level of control risk.

Are not part of the audit evidence considered to support the assertion.

Replace a low assessed level of control risk as audit evidence to support the assertion.

Complement, but do not replace, substantive tests designed to support the assertion.
Ans.
Complement, but do not replace, substantive tests designed to support the assertion.

When an audit is made in accordance with generally accepted auditing standards, the auditor should always

Observe the taking of physical inventory on the balance sheet date.

Obtain certain written representations from management.

Employ analytical procedures as substantive tests to obtain evidence about specific assertions related to
account balances.

Document the understanding of the client’s internal control and the basis for all conclusions about the
assessed level of control risk for financial statement assertions.
Ans.
Obtain certain written representations from management.

What type of opinion should be expressed if the client’s management refuses to provide a representation that
the auditor considers necessary?

Qualified opinion or a disclaimer of opinion.


Qualified opinion or an adverse opinion.
Adverse opinion or a disclaimer of opinion.
Unqualified opinion.
Ans.
Qualified opinion or a disclaimer of opinion.
A written representation from a client’s management that, among other matters, acknowledges responsibility
for the fair presentation of financial statements, should normally be signed by the

Chief financial officer and the chair of the board of directors.


Chief executive officer and the chief financial officer.
Chief executive officer, the chair of the board of directors, and the client’s lawyer.
Chair of the audit committee of the board of directors
Ans.
Chief executive officer and the chief financial officer.

The date of the management representation letter should coincide with the date of the

Statement of Financial Position


Latest related party transaction
Auditor’s report
Latest interim financial information
Ans.
Auditor’s report

Which of the following statements concerning management representations is incorrect?

Representations by management can be a substitute for other audit evidence that the auditor could
reasonably expect to be available.

If the auditor is unable to obtain sufficient appropriate audit evidence regarding a matter, which has, or may
have, a material effect on the financial statements and such audit evidence is expected to be available, this will
constitute a limitation in the scope of the audit, even if a representation from management has been received
on the matter.

If a representation by management is contradicted by other audit evidence, the auditor should investigate the
circumstances and, when necessary, reconsider the reliability of other representations by management.

The auditor’s working papers would ordinarily include a summary of oral discussions with management or
written representations from management.
Ans.
Representations by management can be a substitute for other audit evidence that the auditor could
reasonably expect to be available.

“There have been no communications from regulatory agencies concerning noncompliance with, or
deficiencies in, financial reporting practices that could have a material effect on the financial statements.” The
foregoing passage is most likely from a

A special report.
Letter for an underwriter.
Management representation letter.
Report on internal controls.
Ans.
Management representation letter.

The auditor is required to obtain audit evidence that management


I. Acknowledges its responsibility for the fair presentation of the financial statements in accordance with
applicable financial reporting framework.
II. Has approved the financial statements.

I only.
II only.
Both I and II.
Neither I nor II.
Ans.
Both I and II.

If management refuses to furnish certain written representations that the auditor believes are essential,
which of the following is appropriate?

The auditor can rely on oral evidence relating to the matter as a basis for an unqualified opinion.

The client’s refusal does not constitute a scope limitation that may lead to a modification of the opinion.

This may have an effect on the auditor’s ability to rely on other representations of management.

The auditor should issue an adverse opinion because of management’s refusal.


Ans.
This may have an effect on the auditor’s ability to rely on other representations of management.

Which of the following is responsible for the fairness of the representations made in financial statements?

Client's management
Independent auditor
Audit committee
PICPA
Ans.
Client's management

The primary objective of analytical procedures used in the final review stage of an audit is to

Obtain evidence from details tested to corroborate particular assertions.


Identify areas that represent specific risks relevant to the audit.
Assist the auditor in assessing the validity of the conclusions reached.
Satisfy doubts when questions arise about a client’s ability to continue in existence.
Ans.
Assist the auditor in assessing the validity of the conclusions reached.

Which of the following is true?

Management should evaluate the auditor’s assessment of the entity’s ability to continue as a going concern.

Both management and the auditor should assess the entity’s ability to continue as a going concern
simultaneously.

The auditor should evaluate management’s assessment of the entity’s ability to continue as a going concern.
Going concern assessments will be performed only in exceptional cases.
Ans.
The auditor should evaluate management’s assessment of the entity’s ability to continue as a going concern.

Analytical procedures used in the overall review stage of an audit generally include

Gathering evidence concerning account balances that have not changed from the prior year

Retesting control procedures that appeared to be ineffective during the assessment of control risk

Considering unusual or unexpected account balances that were not previously identified

Performing tests of transactions to corroborate management’s financial statement assertions


Ans.
Considering unusual or unexpected account balances that were not previously identified

An auditor concludes that a substantive auditing procedure considered necessary during the prior period’s
audit was omitted. Which of the following factors would most likely cause the auditor promptly to apply the
omitted procedure?

There are no alternative procedures available to provide the same evidence as the omitted procedure.

The omission of the procedure impairs the auditor’s present ability to support the previously expressed
opinion.

The source documents needed to perform the omitted procedure are still available.

The auditor’s opinion on the prior period’s financial statements was unqualified.
Ans.
The omission of the procedure impairs the auditor’s present ability to support the previously expressed
opinion.

An auditor concludes that the omission of a substantive procedure considered necessary at the time of the
examination may impair the auditor's present ability to support the previously expressed opinion. The
auditor need not apply the omitted procedure if

The risk of adverse publicity or litigation is low.

The results of other procedures that were applied tend to compensate for the procedure omitted.

The auditor's opinion was qualified because of a departure from PFRS.

The results of the subsequent period's tests of controls make the omitted procedure less important.
Ans.
The results of other procedures that were applied tend to compensate for the procedure omitted.

Which of the following is an incorrect statement concerning the relationship of the internal auditor and the
scope of the external audit of an entity’s financial statements?
The external auditor is not required to give consideration to the internal audit function beyond obtaining a
sufficient understanding to identify and assess the risks of material misstatement of the financial statements
and to design and perform further audit procedures.

The internal auditors may determine the extent to which audit procedures should be employed by the
external auditor.

Under certain circumstances, the internal auditors may assist the external auditor in performing substantive
tests and tests of controls.

The nature, timing, and extent of the external auditor’s substantive tests may be affected by the work of
internal auditors.
Ans.
The internal auditors may determine the extent to which audit procedures should be employed by the
external auditor.

The primary reason an engagement letter is obtained by audit firms prior to starting the work is that

it clarifies the responsibilities of management and those of the auditor.


it defines the firm’s policies and procedures regarding new clients.
it provides an insurance policy for companies entering into the agreement.
it communicates the type of opinion that will be rendered on the engagement
Ans.
it clarifies the responsibilities of management and those of the auditor.

Which of the following is not a distinguishing feature of risk-based auditing?

Identifying areas posing the highest risk of financial statement errors.

Analysis of internal control.

Collecting and evaluating evidence.

Concentrating audit resources in those areas presenting the highest risk of financial statement errors.
Ans.
Collecting and evaluating evidence.

The existence of a related party transaction may be indicated when another entity

sells real estate to the corporation at a price that is comparable to its appraised value.
absorbs expense of the corporation.
borrows from the corporation at a rate of interest, which equals the current market rate.
lends to a corporation at a rate of interest, which equals the current market rate.
Ans.
absorbs expense of the corporation.

Which of the following conditions and events may most likely indicate the existence of risks of material
misstatement?

Having personnel with appropriate accounting and financial reporting skills.


Accounting measurements that involve simple processes.
Significant amount of routine or systematic transactions.
Constraints on the availability of capital and credit.
Ans.
Constraints on the availability of capital and credit.

Inherent risk and control risk differ from detection risk in that inherent risk and control risk are

Elements of audit risk while detection risk is not.


Changed at the auditor’s discretion while detection risk is not.
Considered at the individual account-balance level while detection risk is not.
Functions of the client and its environment while detection risk is not.
Ans.
Functions of the client and its environment while detection risk is not.

The process of evaluating the quality of internal control overtime is known as


Risk assessment
Monitoring
Performance reviews
Operational audit
Ans.
Monitoring

Other information that the auditor may use as audit evidence least likely includes

Minutes of meetings.

Confirmations from third parties.

Information obtained by the auditor from such audit procedures as inquiry, observation, and inspection.

Adjustments to the financial statements that are not reflected in formal journal entries.
Ans.
Adjustments to the financial statements that are not reflected in formal journal entries.

S1. The quantity of audit evidence needed is affected by the risk of misstatement and also by the quality of
such audit evidence.
S2. The reliability of audit evidence is influenced by its source and by its nature and is dependent on the
individual circumstances under which it is obtained.

Both statements are true.


Both statements are false.
True; False.
False; True.
Ans.
Both statements are true.

Which of the following pairs of accounts would an auditor most likely analyze on the same working paper?

Notes receivable and interest income.


Accrued interest receivable and accrued interest payable.
Notes payable and notes receivable.
Interest income and interest expense.
Ans.
Notes receivable and interest income.

During the course of an audit engagement an auditor prepares and accumulates audit working papers. The
primary purpose of the audit working papers is to

Aid the auditor planning his work.

Provide a point of reference for future audit engagements.

Support the underlying concepts included in the preparation of the basic financial statements.

Support the auditor's opinion.


Ans.
Support the auditor's opinion.

Which of the following eliminates voluminous details from the auditor’s working trial balance by classifying
and summarizing similar or related items?

Account analyses.
Control accounts
Supporting schedules
Lead schedules.
Ans.
Lead schedules.

The responsibility for the identification and disclosure of related parties and transactions with such parties
rests with the

Auditor
entity’s management
Financial Reporting Standards Council
Securities and Exchange Commission
Ans.
entity’s management

Which of the following statements is true about related party transactions?

In the absence of evidence to the contrary, related party transactions should be assumed to be outside the
ordinary course of business.

An auditor should determine whether a particular transaction would have occurred if the parties had not
been related.

An auditor should substantiate that related party transactions were consummated on terms equivalent to
those that prevail in arm's length transactions.

The audit procedures directed toward identifying related party transactions should include considering
whether transactions are occurring but are not being given proper accounting recognition.
Ans.
The audit procedures directed toward identifying related party transactions should include considering
whether transactions are occurring but are not being given proper accounting recognition.

Why does an auditor document audit evidence?

To comply with the requirements of gathering all available evidence.

To provide client reference for all account balances and correcting entries.

To support audit opinion and to provide evidence that the audit was carried out in accordance with
PSA.

To document all records of misstatements noted in the financial statements.


Ans.
To support audit opinion and to provide evidence that the audit was carried out in accordance with
PSA.

It means the materials prepared by and for, or obtained and retained by the auditor in connection with
the performance of the audit.

Documentation
Audit evidence
Engagement letter
Audit report
Ans.
Documentation

Which of the following conditions constitutes inappropriate working paper preparation?

Flowcharts are included in the working papers.

Findings are cross-referenced to supporting documentation

Tick marks are explained in working papers.

All forms and directives used by the auditee department are included in the working papers.
Ans.
All forms and directives used by the auditee department are included in the working papers.

Which of the following statements concerning evidential matter is true?

Appropriate evidence supporting management’s assertions should be convincing rather than merely
persuasive.

Effective internal control contributes little to the reliability of the evidence created within the entity.

The cost of obtaining evidence is not an important consideration to an auditor in deciding what evidence
should be obtained.

A client’s accounting records cannot be considered sufficient evidence to support the financial statements.
Ans.
A client’s accounting records cannot be considered sufficient evidence to support the financial statements.

You might also like